Medbullets

Pataasin ang iyong marka sa homework at exams ngayon gamit ang Quizwiz!

A 70-year-old man with history of asthma and chronic obstructive pulmonary disease presents to the emergency department with resting dyspnea for the past 2 hours after watching his granddaughter play soccer. Despite albuterol, ipratropium, magnesium, methylprednisolone, and high flow nasal cannula, the patient agrees to elective intubation and is transferred to the intensive care unit. His hospital course is prolonged by pneumonia, which is managed with vancomycin, pipercillin-tazobactam, and gentamicin to cover for ventilator-associated etiologies. He is maintained on nasogastric tube feeds, lactated ringer fluid boluses, and subcutaneous heparin for deep vein thrombosis prophylaxis. On hospital day 7, the patient proves difficult to wean from his ventilator due to labored tachypnea. As the nurse rotates the patient to change his sacral pressure ulcer dressing, the patient's peak airway pressure increases sharply to 55 cm H2O. The nurse noted that the patient was wincing at the time. His temperature is 100.1°F (38.1°C), blood pressure is 100/50 mmHg, and pulse is 100/min. His respirations are 22/min on the ventilator despite a set respiratory rate of 14/min. His laboratory results are shown below: Leukocyte count and differential: Leukocyte count: 18,000/mm^3 Segmented neutrophils: 90% Bands: 10% Eosinophils: 5% Basophils: 2% Lymphocytes: 2% Monocytes: 1% Serum: Na+: 135 mEq/L Cl-: 107 mEq/L K+: 3.2 mEq/L HCO3-: 18 mEq/L Urea nitrogen: 45 mg/dL Glucose: 89 mg/dL Creatinine: 1.3 mg/dL Alkaline phosphatase: 100 U/L Aspartate aminotransferase (AST, GOT): 250 U/L Alanine aminotransferase (ALT, GPT): 300 U/L Gamma-Glutamyltransferase (GGT): 2 U/L Amylase: 10 IU/dL Lipase: 5 IU/dL Bedside ultrasound imaging is shown in Figure A. What is the most likely diagnosis? Acalculous cholecystitis Acute cholangitis Biliary colic Calculous cholecystitis Critical illness pancreatitis

A, Acute Cholecystitis This critically ill patient has persistent fever, vague pain, leukocytosis, and unexplained transaminitis in the setting of a thickened acalculous gallbladder, suggesting a diagnosis of acalculous cholecystitis. Acalculous cholecystitis is acute or chronic cholecystitis in the absence of stones. It is typically due to gallbladder ischemia or stasis, seen particularly in patients with critical illness or those who have undergone prolonged fasting and infrequent use of their gastrointestinal tract. Clinical features include fever, vague RUQ abdominal pain, leukocytosis, occasionally transaminitis, and sometimes jaundice. Ultrasound reveals gallbladder wall thickening and sludge in the absence of stones. Treatment includes broad-spectrum antibiotics and cholecystostomy for the unstable patient. Definitive cholecystectomy can be pursued when the patient is more healthy. Figure A shows an abdominal ultrasound of the gallbladder. Note the increased aeration, thickened walls, and sludge lining, all in the absence of stones. Incorrect Answers: Answer 2: Acute cholangitis is on the differential, but there is no evidence of air in the visible portion of the biliary tract in the ultrasound. Answer 3: Biliary colic alone would not cause the laboratory abnormalities present in this patient. Answer 4: Calculous cholecystitis is not likely in the absence of stones. Answer 5: Critical illness pancreatitis typically presents with significant lipemia, which is absent in this case. Bullet Summary: Acalculous cholecystitis can present in critically ill or prolonged fasting patients with symptoms and signs similar to patients with calculous cholecystitis.

A 67-year-old woman presents from home hospice with a change in her mental status. She has seemed more confused lately and is unable to verbalize her symptoms. Her temperature is 102°F (38.9°C), blood pressure is 117/65 mmHg, pulse is 110/min, respirations are 19/min, and oxygen saturation is 95% on room air. Physical exam is notable for a right upper quadrant mass that elicits discomfort when palpated. Ultrasound is notable for pericholecystic fluid and gallbladder wall thickening without any gallstones. Which of the following is the most likely diagnosis? Acalculous cholecystitis Calculous cholescystitis Choledocholithiasis Emphysematous cholecystitis Hepatic adenoma

A, Acute Cholecystitis This patient is presenting with a right upper quadrant mass and ultrasound findings suggesting a diagnosis of acalculous cholecystitis. Cholecystitis occurs when the gallbladder becomes irritated and inflamed. This is typically due to gallstones which can obstruct the normal flow of bile leading to irritation and inflammation of the gallbladder. In elderly or critically ill patients, cholestasis can lead to biliary sludge and acalculous cholecystitis which presents on ultrasound with a thickened gallbladder wall and pericholecystic fluid without stones. It can progress to gallbladder ischemia, infection, and necrosis. The treatment is keeping the patient NPO, administering IV fluids and antibiotics, and either percutaneous drainage or cholecystectomy. Cholecystectomy would only be indicated once the patient has been medically stabilized. Incorrect Answers: Answer 2: Calculous cholescystitis would present with right upper quadrant pain, a positive Murphy sign, and ultrasound findings including visible stones, gallbladder wall thickening, and pericholecystic fluid. These patients are less critically ill than patients with acalculous cholecystitis or ascending cholangitis. Answer 3: Choledocholithiasis occurs when a stone obstructs the common bile duct causing symptoms including right upper quadrant pain and abnormal lab values including an elevated AST, ALT, and alkaline phosphatase. The diagnosis is typically confirmed with endoscopic retrograde cholangiopancreatography (ERCP) followed by sphincterotomy. An initial ultrasound may show dilation of the gallbladder and common bile duct. Answer 4: Emphysematous cholecystitis is a progression of acute cholecystitis when there is infection of the gallbladder wall with gas producing organisms. Right upper quadrant crepitus may be palpated on exam. Air within the gallbladder lumen can be seen on CT scan and may be demonstrated on ultrasound. Answer 5: Hepatic adenoma presents with a right upper quadrant mass that would be detected on ultrasound and is associated with birth control or anabolic steroid use. It can rupture, leading to hypotension and tachycardia. Bullet Summary: Acalculous cholecystitis presents in an elderly or ill patient with altered mental status, pericholecystic fluid, gallbladder wall thickening, and an absence of stones on ultrasound.

A 29-year-old female is hospitalized 1 day after an endoscopic retrograde cholangiopancreatography (ERCP) because of vomiting, weakness, and severe abdominal pain. Physical examination findings include abdominal tenderness and diminished bowel sounds. A CT scan demonstrates fluid around the pancreas. Serum levels of which of the following are likely to be low in this patient? Calcium Glucose Amylase Lipase Triglycerides

A, Acute Pancreatitis The patient is hospitalized 1 day post-ERCP with symptoms and imaging suggestive of acute pancreatitis. In acute pancreatitis, serum amylase levels are elevated and free digested fats may bind with serum calcium, causing hypocalcemia. ERCP is the third most common cause of acute pancreatitis, following alcohol ingestion and gallstones. Acute pancreatitis symptoms include nausea/vomiting, severe epigastric pain, weakness, low fever, shock, and tetany due to hypocalcemia. Physical exam findings include abdominal tenderness, diminished bowel sounds, and jaundice (rare). Treatment includes pain control and acute management (IV fluids, bowel rest, NG decompression, antibiotics, and oxygen). Illustration A shows an axial CT of acute pancreatitis. Extensive fluid collections surround the pancreas. Incorrect Answers: Answers 2-4: Serum levels of glucose, amylase, and lipase would be expected to increase in acute pancreatitis. Answer 5: Hypertriglyceridemia can cause acute pancreatitis.

A 45-year-old man with a history of biliary colic presents with one-day of intractable nausea, vomiting, and abdominal pain radiating to the back. Temperature is 99.7 deg F (37.6 deg C), blood pressure is 102/78 mmHg, pulse is 112/min, and respirations are 22/min. On abdominal exam, he has involuntary guarding and tenderness to palpation in the right upper quadrant and epigastric regions. Laboratory studies show white blood cell count 18,200/uL, alkaline phosphatase 650 U/L, total bilirubin 2.5 mg/dL, amylase 500 U/L, and lipase 1160 U/L. Which of the patient's laboratory findings is associated with increased mortality? White blood cell count Alkaline phosphatase Total bilirubin Amylase Lipase

A, Acute Pancreatitis This clinical vignette is consistent with acute gallstone pancreatitis. According to Ranson's criteria, an initial white blood cell count greater than 18,000/uL is associated with increased mortality. The most common causes of acute pancreatitis in adults are gallstones and alcohol. Patients may present with nausea, anorexia, and abdominal pain, characteristically in the epigastric region and radiating to the chest and/or back. Elevated lipase is more specific for pancreatitis than elevated amylase since lipase is produced primarily in the pancreas and may remain elevated for several days longer than amylase. As reviewed by Quinlan, acute pancreatitis may be diagnosed based on presence of at least two of the following three criteria: characteristic abdominal pain, amylase and/or lipase at least three times greater than normal levels, and characteristic imaging findings, usually with contrast enhanced CT that may show pancreatic necrosis and inflammation with or without extrapancreatic involvement. Treatment involves bowel rest, aggressive fluid hydration, and pain control. Potential complications from acute pancreatitis include pseudocyst formation, hemorrhage, infection, and multi-organ failure. Given the high morbidity and mortality associated with acute pancreatitis, multiple predictive scoring systems have been reported. A retrospective study by Papachristou et al. demonstrated similar prognostic accuracy among various scoring systems including Ranson's, APACHE-II, CTSI, and BISAP. Ranson's criteria is one of the earliest scoring systems created, based on two assessments made on presentation and 48 hours later. Estimates are based on age, WBC count, glucose, LDH, AST, hematocrit, BUN, calcium, arterial pO2, base deficit, and fluid needs. Incorrect Answers: Answer 2-5: None of these are included in Ranson's criteria or any other predictive scoring systems for estimating mortality from acute pancreatitis. Of note, amylase and lipase are useful for diagnosis, but are not predictive for prognosis.

A 43-year-old homeless man is brought to the emergency room for severe abdominal pain. He is known to the emergency department for repeated visits for alcohol intoxication. He reports that the pain started about 3 hours ago and describes it as sharp, piercing, 10/10, and concentrated at the epigastric region with radiation to the back. His past medical history is unclear given his homeless status. His temperature is 100.4°F (38°C), blood pressure is 104/80 mmHg, the pulse is 104/min, and respirations are 16/min. A physical examination demonstrates epigastric tenderness and the finding shown in Figure A. What is the most likely diagnosis for this patient? Acute pancreatitis Aortic dissection Cholecystitis Esophageal rupture Perforated gastric ulcer

A, Acute Pancreatitis This patient is likely experiencing alcohol-induced acute pancreatitis as demonstrated by his severe epigastric pain with radiation to the back, alcohol abuse history, and Grey Turner sign (purple discoloration of the flank). Acute pancreatitis describes damage and inflammation of the pancreas leading to autodigestion of the organ secondary to the release of pancreatic enzymes. Alcohol is the second most common cause in adults; other etiologies include gallstones (most common), neoplasm, renal disease, trauma, infections, anorexia, and iatrogenic. Patients often present with severe epigastric pain with radiation to the back that is relieved by leaning forward, nausea/vomiting, and low-grade fever. Physical exam findings such as Grey Turner sign and Cullen sign indicates hemorrhagic pancreatitis. Diagnosis is confirmed via abdominal CT and laboratory tests (e.g., amylase and lipase). Treatment involves supportive management and control of pain. Figure A is a clinical image of bluish-purple discoloration of the flank suggestive of hemorrhagic pancreatitis. Incorrect Answers: Answer 2: Aortic dissection is a possible differential of epigastric pain as it classically presents with chest pain that radiates to the back. Physical examination findings include unequal blood pressures in the arms and/or weak/absent pulses. It would not present with the Grey Turner sign. Answer 3: Cholecystitis is a possible differential of epigastric pain although it most commonly presents as right upper quadrant (RUQ) pain that is worse after ingestion. Although gallstones may lead to acute pancreatitis, this patient's clinical presentation is more suggestive of acute pancreatitis. Answer 4: Esophageal rupture is an important cause for acute chest pain. The condition is more common in alcoholics and often follows extensive retching. Physical examination demonstrates crepitus due to the presence of air. Answer 5: Perforated gastric ulcer may also cause severe epigastric pain. Patients would often exhibit peritoneal signs (e.g., guarding and rigidity) due to the presence of stomach contents within the peritoneum. Bullet Summary: Acute pancreatitis most commonly presents as acute epigastric pain that radiates to the back and is relieved by leaning forward; Grey Turner sign and Cullen sign indicate hemorrhagic pancreatitis.

A 33-year-old man presents to the emergency department with severe abdominal pain. He states that it started yesterday and has been gradually worsening. The patient is an alcoholic and drinks roughly 1 to 2 liters of hard alcohol every day. His temperature is 98.5°F (36.9°C), blood pressure is 122/64 mmHg, pulse is 92/min, respirations are 13/min, and oxygen saturation is 99% on room air. Physical exam is notable for epigastric tenderness, and lab values are notable for a lipase of 334 U/L and a hemoglobin of 10.0 g/dL which is unchanged from his baseline. A nasogastric tube is placed, the patient is started on IV fluids and morphine, and he is transferred to the medicine floor. On the patient's first day on the medical floor, physical exam is notable for the finding in Figure A, and his blood pressure is 122/85 mmHg and pulse is 88/min. Which of the following is the best next step in management? CT scan Emergency thoracotomy Exploratory laparoscopy Serial exams and continuous monitoring Transfusion of packed red blood cells

A, Acute Pancreatitis This patient is presenting with pancreatitis (epigastric pain and an elevated lipase) and Gray Turner sign (ecchymosis of the patient's flanks) which should be further characterized with a CT scan of the abdomen. Cullen sign is ecchymosis of the abdomen, and Gray Turner sign is ecchymosis of the flanks. Both of these signs signify late retroperitoneal hemorrhage which can be associated with blunt abdominal trauma, a ruptured ectopic pregnancy, spontaneous bleeding secondary to a coagulopathy, aortic rupture, and pancreatitis. Pancreatitis and subsequent pancreatic hemorrhage can lead to this presentation when blood and breakdown products can track around the inflamed pancreas. The best next step in management when this finding is seen on physical exam in a stable patient is to assess the etiology with a CT scan. If this were a trauma patient who is hemodynamically unstable, then more appropriate initial interventions may include a FAST exam, transfusion of blood products, IV fluids, and exploratory laparoscopy. Figure A demonstrates Gray Turner sign with ecchymosis of the patient's flanks. Incorrect Answers: Answer 2: Emergency thoracotomy could potentially be indicated to incise the pericardium in pericardial tamponade and for manual cardiac massage. The aorta could also be cross-clamped in order to increase perfusion to vital organs. Answer 3: Exploratory laparoscopy might have been indicated if the patient was hypotensive, tachycardic, and the suspected site of bleeding was the abdomen. Intraabdominal bleeding would be suspected in a patient experiencing trauma with a positive FAST exam. Answer 4: Serial exams and continuous monitoring are appropriate management of a patient experiencing abdominal pain of an unclear etiology with initial workup that is not concerning (such as normal labs and a normal CT scan). Though this patient may certainly need monitoring, a CT scan should be performed to further assess the underlying pathology. Answer 5: Transfusion of packed red blood cells would be indicated in a trauma patient with known bleeding or in a patient with a hemoglobin < 7.0 g/dL. Though a repeat CBC should be checked in this patient, transfusing them merely with this clinical exam finding is not indicated. Bullet Summary: Ecchymosis of the flanks signifies late retroperitoneal hemorrhage and should be evaluated with a CT scan in a stable patient.

A 27-year-old man presents to the emergency department with severe abdominal pain. He was at home working when he felt the pain begin and steadily worsen, thus prompting his presentation. The patient has a past medical history of asthma treated with albuterol. His temperature is 98.5°F (36.9°C), blood pressure is 147/88 mmHg, pulse is 90/min, respirations are 14/min, and oxygen saturation is 98% on room air. Physical exam is notable for epigastric pain that seems to be worsened with palpation. Inspection of the patient reveals the finding in Figure A. The patient's gait is unsteady and his breath smells of alcohol. The patient is started on IV fluids and morphine. Which of the following is the best diagnostic test for the underlying etiology of this patient's presentation? Blood lipid levels CAGE questions and blood alcohol level CT scan of the abdomen Serum lipase Ultrasound

A, Acute Pancreatitis This patient is presenting with xanthomas and symptoms suggestive of pancreatitis for which the best diagnostic test of the underlying etiology would be blood lipid levels. Pancreatitis typically presents with epigastric pain that radiates to the back in an alcoholic patient or a patient who has recently eaten a very fatty meal. The diagnosis requires 2 of the following: 1. Epigastric pain that radiates to the back 2. Amylase or lipase > 3 times upper limit of normal 3. Abnormalities on radiography Hypertriglyceridemia is a cause of pancreatitis and is associated with xanthomas on physical exam. The diagnosis of hypertriglyceridemia can be confirmed with blood lipid levels. Figure A is a clinical image of xanthomas which are commonly seen in hypertriglyceridemia. Incorrect Answers: Answer 2: CAGE questions and blood alcohol level could confirm a patient's alcoholism as the cause of their pancreatitis. Despite signs of alcoholism in this patient, given the physical exam finding of xanthomas, hypertriglyceridemia is a more likely cause. Answer 3: CT scan of the abdomen could confirm the diagnosis of pancreatitis; however, the underlying cause of this patient's hypertriglyceridemia would not be further elucidated by a CT scan. Answer 4: Serum lipase could further elucidate the diagnosis of pancreatitis; however, it would not work up the underlying cause of this patient's pancreatitis (hypertriglyceridemia). Answer 5: Ultrasound could confirm the diagnosis of acute cholecystitis, which would present with right upper quadrant pain worsened by a meal with a positive Murphy's finding on physical exam. Bullet Summary: Hypertriglyceridemia is associated with xanthomas and pancreatitis and can be diagnosed with blood lipid levels.

A 23-year-old man presents to the emergency department with bloody vomitus. The patient is an alcoholic and has presented similarly before. He is given ondansetron; however, he continues to vomit. The patient complains of sudden substernal chest pain and dysphagia after another bout of vomiting. His temperature is 99°F (37.2°C), blood pressure is 117/60 mmHg, pulse is 122/min, respirations are 15/min, and oxygen saturation is 99% on room air. Physical exam is notable for an uncomfortable man with subcutaneous emphysema in the patient's neck and supraclavicular areas. Which of the following is the most likely diagnosis? Esophageal rupture Esophageal varices Mallory Weiss syndrome Spontaneous pneumothorax Tension pneumothorax

A, Boerhaave Syndrome This patient is presenting after profuse vomiting with dysphagia and subcutaneous emphysema suggesting a diagnosis of esophageal rupture. Esophageal rupture typically occurs after surgical procedures or interventions (such as endoscopy) or after profuse vomiting. Patients will complain of neck pain, chest pain, and dysphagia. Other findings include subcutaneous emphysema and a pleural effusion (positive for amylase). The best initial test for suspected esophageal rupture is a gastrografin swallow to confirm the diagnosis and demonstrate the location of the rupture. Surgical repair can occur subsequently. Incorrect Answers: Answer 2: Esophageal varices present with bloody vomitus in a patient with cirrhosis which would present with ascites, jaundice, and lab abnormalities reflecting impaired hepatic function (such as a high INR or a low albumin). Answer 3: Mallory Weiss syndrome presents with bloody vomitus after profuse vomiting; however, it would not present with subcutaneous emphysema which is suggestive of an esophageal rupture. Answer 4: Spontaneous pneumothorax presents with pleuritic chest pain, typically in a tall man. If small, the pneumothorax will resorb spontaneously with oxygen administration. Answer 5: Tension pneumothorax presents after trauma with signs of tension physiology including hypotension, tachycardia, jugular venous distension, and an absence of unilateral breath sounds. Bullet Summary: Esophageal rupture presents after profuse vomiting with retrosternal chest pain, dysphagia, and subcutaneous emphysema.

A 56-year-old man is brought to the emergency room for severe chest pain over the past hour. He reports 10/10, burning pain that is retrosternal and concentrated around the mid-chest region. His wife claims that he has been throwing up for the past several hours from what seems like a "stomach bug," with the latest 2 episodes of emesis being bright red. The patient denies trauma, fever, recent infections, urinary changes or diarrhea but endorses lightheadedness and shortness of breath. His past medical history is significant for hypertension and diabetes. A computed tomography (CT) with IV and oral contrast of the chest and abdomen is obtained and its results are shown in Figure A. What is the most specific physical examination findings for this patient's diagnosis? Crepitus on palpation of the chest Diminished breath sounds Friction rub Pulsus paradoxus Unequal blood pressure in both arms

A, Boerhaave Syndrome This patient likely has Boerhaave syndrome as demonstrated by his severe chest pain and positive CT findings in the setting of prolonged emesis. Patients with the condition may have crepitus on palpation of the chest due to subcutaneous emphysema. Boerhaave syndrome is a medical emergency characterized by a transmural, usually distal esophageal rupture with pneumomediastinum. It results from a sudden increase in intraesophageal pressure combined with negative intrathoracic pressure (e.g., severe straining or vomiting). The condition usually occurs in patients with a normal underlying esophagus. Patients often present with sudden, excruciating retrosternal chest pain with a history of severe retching and vomiting. Physical examination may demonstrate crepitus and Hamman sign (mediastinal crackling with heartbeat upon auscultation). Diagnosis is confirmed via imaging (e.g., CT, radiograph, or water-soluble contrast esophagram). Management is dependent on the severity of the rupture and may include both medical and/or surgical treatments. Figure/Illustration A is a CT image of the chest demonstrating air surrounding the esophagus (arrowhead) and extraluminal contrast (arrow) suggestive of esophageal perforation. Incorrect Answers: Answer 2: Diminished breath sounds can be seen in cases of pneumothorax where the air is present within the pleural space. Although patients may complain of chest pain, the CT findings in this patient are suggestive of Boerhaave syndrome. Answer 3: Friction rub is a finding commonly found in patients with pericarditis. The chest pain in pericarditis is rarely sudden and severe like the one seen in this patient. Answer 4: Pulsus paradoxus is a characteristic finding in cardiac tamponade, severe asthma, and severe chronic obstructive pulmonary disease (COPD). This patient's history and medical history do not suggest these diseases. Answer 5: Unequal blood pressure in both arms suggests aortic dissection, which can also present with sudden, severe chest pain. This patient's medical history of hypertension also makes this diagnosis more likely. However, this patient's imaging finding suggests Boerhaave syndrome. Bullet Summary: Crepitus on palpation of the chest is a physical examination finding suggestive of Boerhaave syndrome.

A 59-year-old man presents to the emergency department with abdominal pain. He states that it seemed to come on abruptly, though he does endorse feeling pain the past several days. He has been experiencing bloody diarrhea as well as a subjective fever. His past medical history is notable for an ST elevation myocardial infarction (STEMI) treated 1 year ago, an abdominal aortic aneurysm (AAA) treated 1 week ago, chronic obstructive pulmonary disease (COPD), asthma, obesity, hypertension, stroke treated 5 years ago, an atrial arrhythmia, diabetes, constipation, and post-traumatic stress disorder (PTSD). His temperature is 102.0°F (39°C), blood pressure is 197/128 mmHg, pulse is 135/min, respirations are 22/min, and oxygen saturation is 92% on room air. On physical exam, you note an obese man in distress. Cardiopulmonary exam is notable for bilateral wheezes and a systolic murmur. There is bilateral lower extremity pitting edema and venous stasis ulcers. Abdominal exam reveals diffuse tenderness which the patient describes as 10/10 in severity. Laboratory values are ordered as seen below. Hemoglobin: 14 g/dL Hematocrit: 42% Leukocyte count: 15,500/mm^3 with normal differential Platelet count: 199,000/mm^3 Serum: Na+: 139 mEq/L Cl-: 100 mEq/L K+: 4.4 mEq/L HCO3-: 22 mEq/L BUN: 32 mg/dL Glucose: 189 mg/dL Creatinine: 1.9 mg/dL Ca2+: 10.2 mg/dL Lactate: 4.5 mg/dL AST: 12 U/L ALT: 10 U/L Which of the following is the most likely explanation for this patient's presentation? Abdominal aortic aneurysm repair complication Atheromatous plaque rupture Atrial fibrillation Obstruction of the small bowel Ruptured appendix

A, Ischemic Colitis This patient is presenting with symptoms of mesenteric ischemia status post abdominal aortic aneursym (AAA) repair suggestive of a diagnosis of an AAA treatment-related complication. AAA typically presents with a tender, pulsatile abdominal mass that can be the patient's chief complaint or can be discovered incidentally on ultrasound/CT. Treatment for an unstable AAA can involve graft placement which can obstruct vessels that supply the bowel such as the inferior mesenteric artery (IMA). This can then lead to symptoms of mesenteric ischemia which can include severe abdominal pain (classically out of proportion to exam), bloody diarrhea, a leukocytosis, and an elevated lactate. It can also lead to ischemic colitis which would present with abdominal pain (not out of proportion to the exam) and bloody stools as well as an elevated lactate. Incorrect Answers: Answer 2: Atheromatous plaque rupture is the typical underlying pathophysiology for mesenteric ischemia. This patient is presenting with symptoms suggestive of mesenteric ischemia; however, in the setting of a recent AAA repair there is another more likely diagnosis. Answer 3: Atrial fibrillation could propagate a clot that could obstruct blood flow leading to this patient's presentation; however, this is a less likely diagnosis in the setting of an acute AAA repair. Answer 4: Obstruction of the small bowel could lead to abdominal pain, distension, nausea, and vomiting; however, it would not cause the severe symptoms this patient is presenting with nor would it cause the laboratory abnormalities noted (unless the obstructed bowel became ischemic). Answer 5: Ruptured appendix could cause diffuse abdominal pain and the laboratory values seen in this patient; however, appendicitis typically occurs in a younger population and this patient has other risk factors that suggest another diagnosis. Bullet Summary: AAA repair can obstruct blood flow to the colon causing ischemic colitis/mesenteric ischemia.

A 71-year-old Caucasian male presents to your office with bloody diarrhea and epigastric pain that occurs 30 minutes after eating. He has lost 15 pounds in 1 month, which he attributes to fear that the pain will return following a meal. He has a history of hyperlipidemia and myocardial infarction. Physical exam and esophagogastroduodenoscopy are unremarkable. What is the most likely cause of this patient's pain? Atherosclerosis Peptic ulcer disease Crohn's disease Amyloid deposition Diverticulosis

A, Ischemic Colitis This patient's presentation is consistent with ischemic colitis secondary to an atherosclerotic process. Ischemic colitis is caused by changes in systemic circulation, narrowing of blood vessels, or a blood clot that results in a loss of blood flow to the large intestine. Most commonly, ischemic colitis occurs at the splenic flexure, a "watershed" area supplied by both the superior and inferior mesenteric arteries. Patients often have a history of hyperlipidemia and cardiovascular disease, and may present with mesenteric angina with meals, secondary to athersclerotic lesions of superior mesenteric artery. Gandhi et al. review the diagnosis and treatment of ischemic colitis. They report that ischemic colitis is the most common form of gastrointestinal ischemia, which typically presents with acute abdominal pain and bloody diarrhea. Yasuhara clarifies the difference between mesenteric angina and ischemic colitis in which there are three major categories based on its clinical features: acute mesenteric ischemia (AMI), chronic mesenteric ischemia (intestinal angina), and colonic ischemia (ischemic colitis). See "Mesenteric Ischemia" topic for clarification. Illustration A demonstrates ischemic colitis as confirmed by a barium study, which shows "thumb-printing" of the colonic mucosa secondary to edema. Incorrect answers: Answer 2: Peptic ulcer disease would likely show findings on an upper endoscopy (esophagogastroduodenoscopy). Answer 3: Post-prandial pain and fear of eating in the setting of known atherosclerotic disease are more consistent with ischemic colitis than with Crohn's disease. Answer 4: Amyloid deposition results in amyloidosis, which does not typically present with post-prandial pain. Answer 5: Diverticulosis can present with painless bright red rectal bleeding (hematochezia), but is often asymptomatic.

A 74-year-old man presents to the emergency room with abdominal pain. He reports acute onset of left lower quadrant abdominal pain and nausea three hours prior to presentation. The pain is severe, constant, and non-radiating. He has had two maroon-colored bowel movements since the pain started. His past medical history is notable for hypertension, hyperlipidemia, atrial fibrillation, insulin-dependent diabetes mellitus, and rheumatoid arthritis. He takes lisinopril, hydrochlorothiazide, atorvastatin, dabigatran, methotrexate. He has a 60 pack-year smoking history and drinks 1-2 beers per day. He admits to missing some of his medications recently because he was on vacation in Hawaii. His last colonoscopy was 4 years ago which showed diverticular disease in the descending colon and multiple sessile polyps in the sigmoid colon which were removed. His temperature is 100.1°F (37.8°C), blood pressure is 145/85 mmHg, pulse is 100/min, and respirations are 20/min. On exam, he has notable abdominal distention and is exquisitely tender to palpation in all four abdominal quadrants. Bowel sounds are absent. Which of the following is the most likely cause of this patient's condition? Cardiac thromboembolism Duodenal compression Perforated intestinal mucosal herniation Paradoxical thromboembolism Splanchnic vasoconstriction

A, Mesenteric Ischemia The patient in this vignette presents with acute onset abdominal pain, melena, absent bowel sounds, and a history of thromboembolic disease with medication non-adherence suggestive of mesenteric ischemia. Mesenteric ischemia is commonly caused by acute arterial occlusion from a cardiac embolism. Mesenteric ischemia refers to an abrupt reduction in perfusion to the intestinal circulation leading to intestinal ischemia. This presents clinically as acute onset abdominal pain, nausea, vomiting, and melena or hematochezia. The most common causes of mesenteric ischemia, in order, are cardiac embolism due to atrial fibrillation or valvular disease, mesenteric atherosclerosis, vasoconstriction in the setting of severe hypotension, and venous thrombosis. Incorrect Answers: Answer 2: Duodenal compression occurs in superior mesenteric artery (SMA) syndrome due to a decrease in the angle between the SMA and aorta. Clinically, this may present with nausea, vomiting, weight loss, and post-prandial epigastric pain. Answer 3: Diverticulitis refers to infection and macroperforation of colonic diverticuli (mucosal outpouchings) found throughout the colon. Clinically, diverticulitis presents with left-sided abdominal pain, fever, and either diarrhea or constipation. Answer 4: Paradoxical thromboembolism refers to a venous thrombus traversing a patent foramen ovale to enter the arterial system. In rare cases, a venous thrombus can use this route and lodge into the mesenteric arterial supply, leading to mesenteric ischemia. Answer 5: Splanchnic vasoconstriction is a naturally occurring process in the setting of severe hypotension (i.e., shock or hypovolemia) to maintain adequate blood pressure. This can lead to mesenteric ischemia by depriving the downstream intestinal tissue of sufficient blood supply. Bullet Summary: Mesenteric ischemia is most commonly caused by a dislodged thrombus, frequently of cardiac origin, that migrates to the mesenteric vasculature.

A 67-year-old man presents to the emergency department with abdominal pain. The patient states that whenever he eats he gets diffuse abdominal pain and experiences nausea, vomiting, and diarrhea. He states that the pain is mostly in the epigastric region. As a result of the pain, the patient has begun to eat less and has lost 10 pounds recently. The patient's past medical history is notable for diabetes, hypertension, dyslipidemia, and a 30 pack-year smoking history. His temperature is 99.2°F (37.3°C), blood pressure is 150/95 mmHg, pulse is 72/min, respirations are 16/min, and oxygen saturation is 100% on room air. Physical exam is notable for a nontender abdomen. The patient states that his abdominal pain mostly resolved by the time he arrived to the emergency department. Which of the following is the pathophysiology of the underlying diagnosis? Atherosclerotic vessel narrowing Common bile duct obstruction Gastric mucosa erosion Pancreatic enzyme activation Thromboembolism

A, Mesenteric Ischemia This patient is presenting with abdominal pain that is associated with eating as well as nausea, vomiting, and diarrhea, concerning for a diagnosis of chronic mesenteric ischemia which is associated with atherosclerotic vessel narrowing. Chronic mesenteric ischemia occurs secondary to narrowing of the vessels of the gut secondary to atherosclerosis and injury from inflammation (such as smoking). Patients will typically present with diffuse abdominal pain whenever they eat and is associated with symptoms including nausea, vomiting, and diarrhea. The patient's abdomen tends to be nontender (because the pain is secondary to reduced blood flow from vessel disease), and patients tend to endorse weight loss from food aversion. Treatment may involve rehydration, antiemetics, small meals, tight blood glucose and lipid control, and vascular surgery (including angioplasty, stenting, and surgical revascularization). Incorrect Answers: Answer 2: Common bile duct obstruction leading to swelling and pericholecystic fluid could be seen in acute cholecystitis (secondary to cholelithiasis). Cholecystitis presents in a fat, fertile, female in her forties with right upper quadrant and colicky abdominal pain that is worsened by eating with a positive Murphy sign. Answer 3: Gastric mucosa erosion describes a peptic ulcer that presents with a more constant and gnawing pain that can be worsened by eating. It would not present with symptoms only associated with eating with diarrhea after meals. Answer 4: Pancreatic enzyme activation and inflammatory damage to the pancreas is the pathophysiology of pancreatitis. Acute pancreatitis would present with epigastric pain in an alcoholic or after a fatty meal. Laboratory studies would show an elevated lipase. Answer 5: Thromboembolism describes acute mesenteric ischemia where a blood clot has been dislodged and suddenly completely occludes the flow of blood to a portion of the colon. This presents with sudden onset and severe abdominal pain with the classic "pain out of proportion to exam." Bullet Summary: Chronic mesenteric ischemia occurs secondary to atherosclerosis and vessel narrowing and presents with abdominal pain that is associated with nausea, vomiting, and diarrhea when the patient eats.

A 55-year-old man presents to the emergency department with nausea and vomiting. The patient states that he has felt nauseous for the past week and began vomiting last night. He thought his symptoms would resolve but decided to come in when his symptoms worsened. He feels that his symptoms are exacerbated with large fatty meals and when he drinks alcohol. His wife recently returned from a cruise with symptoms of vomiting and diarrhea. The patient has a past medical history of poorly managed diabetes, constipation, anxiety, dyslipidemia, and hypertension. His temperature is 99.5°F (37.5°C), blood pressure is 197/128 mmHg, pulse is 100/min, respirations are 17/min, and oxygen saturation is 95% on room air. Physical exam reveals a systolic murmur heard loudest along the left upper sternal border. Abdominal exam reveals an obese, tympanitic and distended abdomen with a 3 cm scar in the right lower quadrant. Vascular exam reveals weak pulses in the lower extremities. Which of the following is the most likely diagnosis? Adhesions Enteric nervous system damage Impacted stool Norovirus Twisting of the bowel

A, Small Bowel Obstruction This patient is presenting with symptoms suggestive of a small bowel obstruction which typically occurs secondary to adhesions. Small bowel obstructions (SBO) typically present with an absence of bowel sounds, bowel movements, and flatus, as well as abdominal distension, nausea, and vomiting. They typically occur secondary to adhesions which entrap the bowel and disrupt its normal peristalsis. Adhesions typically occur after abdominal surgery or trauma as part of the normal healing process. Incorrect Answers: Answer 2: Enteric nervous system damage describes diabetic gastroparesis which can present similarly to SBO; however, this patient's age and history of surgery points toward SBO as the most likely diagnosis. Answer 3: Impacted stool describes constipation which could present similarly to SBO; however, this diagnosis is less likely to cause nausea and vomiting unless it is severe. The vomit typically smells like stool. Answer 4: Norovirus typically causes nausea, vomiting, and diarrhea but does not cause distension and a tympanitic abdomen. Answer 5: Twisting of bowel describes a volvulus which presents similarly to SBO; however, it is a more common diagnosis in the elderly and is a less common diagnosis than is SBO in a patient with a history of abdominal surgery. Bullet Summary: Small bowel obstruction presents with nausea, vomiting, absence of flatus/bowel movements, and a distended/tympanitic abdomen.

A 73-year-old woman with a history of diabetes presents to the emergency department with severe abdominal pain and constipation for 24 hours. Vital signs are T: 100.3 F, HR: 80 bpm, BP: 143/82 mmHg, RR: 24/min, O2 Sat: 95%. Abdominal exam is notable for diffuse abdominal tenderness to palpation, without rebound or guarding. Laboratory studies are notable for leukocytosis, though lactic acid is within normal limits. Abdominal x-ray is shown in Figure A. Which of the following are the best next management steps for this patient? Sigmoidoscopy and placement of a rectal tube for 48-72 hours Barium Enema with abdominal xray Immediate laparotomy with Hartmann's procedure Immediate laparotomy with sigmoidectomy and primary anastomosis NPO, NG tube decompression and neostigmine

A, Volvulus This elderly patient presents with signs and symptoms of sigmoid volvulus (SV), and requires prompt derotation and decompression with a sigmoidoscopy or colonoscopy, followed by rectal tube placement and observation for signs of bowel ischemia. SV is the third leading cause of colonic obstruction in adults. It manifests as a result of redundant sigmoid loops, which can rotate about their narrow mesentery leading to obstruction and ischemia. Initial management of SV is as described above, with definitive surgery following initial successful decompression. Detorsion and decompression is successful in well over 50% of patients, allowing for time to electively operate on the patient. Once detorsed, sigmoid resection with primary anastomosis is the procedure of choice, though more extensive procedures may be required, depending on the extent of colonic damage. Detorsion should be attempted in patients who present with no signs of rupture (i.e. peritonitis), and who are hemodynamically stable. Figure A shows an abdominal x-ray demonstrating dilated loops of large bowel, with characteristic coffee bean sign, concerning for sigmoid volvulus. Incorrect Answers: Answer 2: A barium enema is not indicated in this patient, as a diagnosis is readily made via abdominal x-ray. Answer 3: Immediate laparotomy with hartmann's procedure is not the management of choice for sigmoid volvulus, especially given that this patient shows no signs of hemodynamic instability. Answer 4: Though sigmoidectomy and primary anastomosis is the procedure of choice, detorsion and decompression via sigmoidoscopy should be attempted first. Answer 5: NPO, NG tube decompression, and neostigmine are possible treatments for colonic pseudo-obstruction (Ogilvie's Syndrome).

A 1-month-old infant girl is brought to the emergency department by her parents for continuous, projectile emesis for 6 hours. The patient was born at 37 weeks gestation without any complications. She is up to date on her vaccinations and has no medical history. The mother states that the patient began throwing up after her feed this morning and has been extremely fussy and inconsolable. The vomitus was initially white but is now green in color. The parents deny any clear precipitating factor, fever, abnormal ingestion, or any recent sick contacts but endorse an episode of hematochezia an hour ago. A physical examination demonstrates dry mucous membranes and diffuse abdominal tenderness and guarding. A radiograph of the abdomen is obtained and shown in Figure A. What is the most likely diagnosis? Intestinal malrotation Intussusception Necrotizing enterocolitis Pyloric stenosis Viral infection

A, Volvulus This patient likely has intestinal malrotation with volvulus as demonstrated by bilious emesis, abdominal tenderness, and positive radiograph findings (i.e., prominent loops of gas-filled small bowel). Intestinal malrotation is a congenital anomaly that results from an arrest of normal rotation of the midgut during development. Anomalies may include small intestine that is found predominantly on the right side of the abdomen, displaced cecum, absent or displaced ligament of Treitz, or narrow mesentery. These abnormalities may lead to volvulus, which is when a loop of intestine twists around itself and the mesentery, resulting in bowel obstruction. Patients commonly present with bilious vomiting, abdominal pain, abdominal distension, and sometimes hematochezia. However, it is worth noting that patients without acute volvulus may be asymptomatic. Bilious vomiting in a child younger than 1 year of age should immediately raise suspicion for malrotation with possible volvulus. Diagnosis usually begins with an abdominal radiograph to rule out perforation and is confirmed via an upper GI series. Treatment involves surgery with the Ladd procedure. In a patient with suspected volvulus, immediate laparotomy with surgical exploration is indicated. Figure/Illustration A is an abdominal radiograph with several air-filled loops of the small bowel (circle) and no gas at the rectum suggestive of intestinal obstruction (arrow). Incorrect Answers: Answer 2: Intussusception is a condition in which 1 segment of intestine telescopes inside of another segment. Patients can present with similar symptoms such as vomiting, abdominal pain, and bloody stool. However, these "attacks" are more intermittent rather than a constant period of projectile emesis. Answer 3: Necrotizing enterocolitis is a medical condition where a portion of the bowel dies. Typical symptoms often include feeding intolerance, failure to thrive, abdominal distension, and bloody stools. However, this condition is commonly seen in premature infants and is uncommon in an otherwise healthy 1-month-old infant. Answer 4: Pyloric stenosis describes the narrowing of the pylorus. Symptoms often present within the first week to month of life with progressively worsening, often projectile vomiting. Episodes commonly follow feeds and are usually non-bilious. This patient's bilious projectile vomiting is more suggestive of intestinal malrotation. Answer 5: Viral infection can lead to emesis. However, enteric viral infections often present with both emesis and diarrhea. This patient's lack of sick contacts and the presence of bilious emesis should raise suspicion for intestinal malrotation. Bullet Summary: Bilious emesis in a child younger than 1 year of age should raise suspicion for intestinal malrotation with volvulus.

A 17-year-old boy is brought to the emergency room by his parents for increasing abdominal pain. He started having periumbilical pain last night but now reports increasing right lower quadrant pain. He reports that about an hour ago, the pain acutely intensified and was not responsive to ibuprofen or acetaminophen anymore. On physical exam, his abdomen is rigid with involuntary guarding and tenderness to light percussion. A right lower quadrant ultrasound shows an enlarged appendix with possible perforation. He is emergently taken to the operating room.

Acute Abdomen (snapshot)

A 30-year-old G0P0 woman presents to the emergency room for acute onset abdominal pain and nausea. She recently had unprotected sex with 2 different partners. On physical exam, she has involuntary guarding and tenderness to percussion. A urine pregnancy test is positive, and a transvaginal ultrasound shows a ruptured ectopic pregnancy.

Acute Abdomen (snapshot)

A 40-year-old obese woman presents to the emergency room for pain in her right upper quadrant of her abdomen. She reports that she has had similar pain on and off for the past few weeks, but this time the pain has persisted for over an hour. She also reports nausea, vomiting, and loss of appetite. On physical exam, she has right upper quadrant pain and inspiratory arrest with deep palpation of the area. An ultrasound of that area reveals distended gallbladder with thickened gallbladder wall and gallstones. She is admitted for further management and the general surgery team is consulted for possible surgery within the next 72 hours.

Acute Cholecystitis (snapshot)

A 45-year-old obese woman presents to the emergency room with steady, severe, aching pain in the upper right quadrant that radiates to the right scapula. The onset was acute and occurred 30 minutes after lunch. She also reports nausea with vomiting. On physical exam, it is noted that she stops breathing on deep palpation of the right quadrant. Laboratory studies show an absolute neutrophillic leukocytosis with a left shift.

Acute Cholecystitis(snapshot)

A 15-year-old boy with severe nodulocystic acne presents to the emergency room for sudden onset epigastric pain radiating to the back, as well as nausea and vomiting. An abdominal computed tomography shows findings concerning for pancreatitis. On further chart review, the physician discovers that he has been taking isotretinoin, and the dermatologist had been monitoring his triglycerides as they were mildly elevated at the last clinical visit. However, the patient reports that he had not only doubled his dose in the last week in an attempt to speed up the process but also had been eating burgers and fries for every meal. Laboratory results show significantly elevated levels of triglycerides in the blood.

Acute Pancreatitis (snapshot)

A 50-year-old man presents to the emergency room with acute onset epigastric pain. His past medical history includes hyperlipidemia, hypertriglyceridemia, diabetes, and alcohol abuse. His last drink was this morning and he drinks 20 beers a day. He denies any fevers or chills but reports nausea and 2 episodes of nonbloody, nonbilious vomiting. Physical exam is notable for tenderness to palpation of the epigastrium. There is no muscle spasm with percussion of the cheeks. Laboratory evaluation shows significantly elevated amylase and lipase. He is started on aggressive fluid resuscitation.

Acute Pancreatitis (snapshot)

A 24-year-old woman presents to the emergency room with abdominal pain for the past 6 hours. The pain was originally located around the umbilical area but has since migrated to the right lower quadrant (RLQ). It is described as a 8/10, stabbing pain that is worse with movement. Her past medical history is unremarkable. A physical examination demonstrates tenderness of the RLQ with moderate guarding.

Appendicitis (snapshot)

A 46-year-old man presents to the emergency department with nausea, vomiting, constipation for the past 2 days. He also complains of abdominal pain that initially started at the periumbilical area then traveled to the right lower quadrant. He is tachycardic, febrile, and mildly hypertensive on examination. A physical examination demonstrates a diffusely tender abdominal with rebound, especially at the right lower quadrant.

Appendicitis(snapshot)

A patient presents to the emergency department with abdominal pain. While having dinner, the patient experienced pain that prompted the patient to come to the emergency department. The patient states that the pain is episodic and radiates to the shoulder. The patient's temperature is 98°F (36.7°C), blood pressure is 120/80 mmHg, pulse is 80/min, respirations are 13/min, and oxygen saturation is 98% on room air. Laboratory values are ordered and return as below. Hemoglobin: 12 g/dL Hematocrit: 36% Leukocyte count: 4,500 cells/mm^3 with normal differential Platelet count: 247,000/mm^3 Serum: Na+: 140 mEq/L Cl-: 100 mEq/L K+: 4.6 mEq/L HCO3-: 24 mEq/L BUN: 15 mg/dL Glucose: 90 mg/dL Creatinine: 0.8 mg/dL Ca2+: 10.0 mg/dL AST: 11 U/L ALT: 11 U/L On physical exam, the patient demonstrates abdominal tenderness that is most prominent in the right upper quadrant. Which of the following represents the most likely demographics of this patient? A middle-aged male with a positive urea breath test A middle-aged overweight mother A middle-aged patient with a history of bowel surgery An elderly diabetic with vascular claudication An elderly smoker with painless jaundice

B, Acute Cholecystitis This patient is presenting with right upper quadrant abdominal pain that radiates to the shoulder and is associated with eating, suggestive of a diagnosis of acute cholecystitis. The most likely demographic to accompany this presentation is a middle-aged overweight mother. Acute cholecystitis presents with right upper quadrant abdominal pain that is episodic and radiates to the shoulder. Typically, this occurs after a patient has consumed a fatty meal or alcohol. The classic patient demographic is a Fat, Fertile, Female in her Forties. The diagnosis should be confirmed with an ultrasound, the patient should be made NPO and started on IV fluids, and a cholecystectomy should be performed once the episode ends. Incorrect Answers: Answer 1: A middle-aged male with a positive urea breath test describes the risk factors for gastric ulcer disease which presents with gastric pain that is worse with meals. These patients will often avoid eating due to the pain associated with meals. Answer 3: A middle-aged patient with a history of bowel surgery describes bowel entrapment due to adhesions. These patients would experience colicky abdominal pain, nausea, and vomiting. Answer 4: An elderly diabetic with vascular claudication describes the type of patient that would present with mesenteric ischemia. Mesenteric ischemia presents with pain out of proportion to exam findings that is worsened with meals. Answer 5: An elderly smoker with painless jaundice describes the type of patient who would present with pancreatic cancer. Bullet Summary: Acute cholecystitis presents in a Fat Female who is Fertile in her Forties.

A 44-year-old woman presents to the emergency department with abdominal pain. This symptom started yesterday and has been gradually worsening. The patient has a past medical history of acute cholecystitis and had her gallbladder removed 1 month ago. Her temperature is 97.7°F (36.5°C), blood pressure is 144/84 mmHg, pulse is 89/min, respirations are 12/min, and oxygen saturation is 98% on room air. Physical exam is notable for right upper quadrant tenderness to palpation. An ultrasound is performed demonstrating a dilated common bile duct and an absence of a gallbladder. Laboratory studies are ordered as seen below. Hemoglobin: 13 g/dL Hematocrit: 37% Leukocyte count: 9,500/mm^3 with normal differential Platelet count: 168,000/mm^3 Serum: Na+: 139 mEq/L Cl-: 100 mEq/L K+: 4.8 mEq/L HCO3-: 25 mEq/L BUN: 23 mg/dL Glucose: 99 mg/dL Creatinine: 1.2 mg/dL Ca2+: 10.2 mg/dL AST: 82 U/L ALT: 100 U/L Alkaline phosphatase: 250 U/L Which of the following is the most appropriate next step in management? CT abdomen/pelvis Endoscopic retrograde cholangiopancreatography Laparoscopy MRI abdomen/pelvis Pain control and reassurance

B, Acute Cholecystitis This patient is presenting with right upper quadrant abdominal pain, an ultrasound demonstrating a dilated common bile duct, and labs demonstrating and elevated AST, ALT, and alkaline phosphatase. In the setting of her recent cholecystectomy, a diagnosis of postcholecystectomy syndrome is suspected, and further workup is needed with endoscopic retrograde cholangiopancreatography. Postcholecystectomy syndrome presents after cholecystectomy with similar symptoms of cholecystitis including right upper quadrant pain/discomfort. The most appropriate initial step in management is an ultrasound demonstrating a dilated common bile duct. Laboratory tests can further support the diagnosis and will demonstrate elevated AST, ALT, and alkaline phosphatase. If the diagnosis is suspected, further workup is indicated and could include magnetic resonance cholangiopancreatography (MRCP), endoscopic retrograde cholangiopancreatography (ERCP), or endoscopic ultrasound. Treatment should target the underlying cause such as a retained stone or sphincter of Oddi dysfunction. Incorrect Answers: Answer 1: CT abdomen/pelvis would not add additional useful detail to this patient's diagnosis given the ultrasound findings. After an ultrasound demonstrating common bile duct dilation a more accurate test such as ERCP and MRCP is needed. Answer 3: Laparoscopy is performed to search for an etiology of a patient's symptoms (typically abdominal or pelvic pain) and diagnose the pathology and possibly correct it. For example, endometriosis (caused by retrograde menstruation causing dysmenorrhea, dyschezia, and dyspareunia) can be diagnosed and treated with laparoscopy to localize the lesions and ablation. Answer 4: MRI abdomen/pelvis would be an inappropriate further test to characterize and treat this patient's postcholecystectomy pain. It could be appropriate in diagnosing appendicitis (periumbilical pain that migrates to the right lower quadrant) in a pregnant patient or child to avoid radiation exposure. Answer 5: Pain control and reassurance are inappropriate as this patient may have a retained stone or sphincter of Oddi dysfunction which requires further diagnostic testing and treatment. Bullet Summary: Postcholecystectomy syndrome requires confirmation of the diagnosis with endoscopic retrograde cholangiopancreatography (ERCP), magnetic resonance cholangiopancreatography (MRCP), or endoscopic ultrasound.

A 39-year-old male presents to the emergency department with acute abdominal pain and anorexia for two days. The patient has an extensive history of alcoholism with a hospitalization four months ago after being brought in by the police department for intoxication. On physical exam vital signs are T 99.0 F, BP 100/68 mmHg, HR 135 bpm, RR 20 rpm, and O2 Sat 100%. Abdominal exam is notable for exquisite tenderness to palpation over the epigastrium and the following finding (Figure A). Therapy for this patient should consist of which of the following? Endoscopic retrograde cholangiopancreatography (ERCP) IV fluid hydration, analgesia, and intensive care management Treat with chlordiazepoxide and intensive care management Esophagogastroduodenoscopy (EGD) with variceal banding Percutaneous transhepatic cholangiography (PTC)

B, Acute Pancreatitis This patient with a history of alcohol abuse is presenting with acute epigastric tenderness, vital sign instability, and Gray Turner's sign suggestive of hemorrhagic acute pancreatitis. Management with IV fluids and supportive therapy in the intensive care unit is appropriate. Acute pancreatitis is a potentially fatal condition most commonly caused by alcohol ingestion or obstruction of the pancreatic duct by a gallstone (gallstone pancreatitis). Pathogenesis involves inflammation of the pancreatic parenchyma and activation of trypsinogen into trypsin resulting in auto-digestion of pancreatic and surrounding tissue. Other causes of pancreatitis include blunt abdominal trauma, auto-immune mediated pancreatitis, infection, and drug induced. Carroll et al. discuss the management of acute pancreatitis, including risk stratification of patients. Stratification through APACHE II Scale or CT Severity Index can provide invaluable insight into patient morbidity and mortality and help guide escalation of care. In the U.S. just over 200,000 hospitalizations occur yearly due to acute pancreatitis, while mortality remains low in moderate uncomplicated cases, those with necrosis can be as high as 10-30% depending on super infection. An area of controversy in the management of acute pancreatitis is the approach of nutrition for the patient. Mirtallo et al. discuss the evidence for enteral nutrition on patients with acute pancreatitis. Mounting evidence supports the use of nutritional support in patients with acute pancreatitis, with enteral nutrition. Even with this evidence, there remains a prevailing clinical practice to withhold enteral nutrition, in the belief that bowel rest is advantageous. However, a number of recommendations support the use of enteral nutrition over parenteral nutrition, unless there is a contraindication to enteral feeds. Figure A shows a photograph of a patient with blood tracking along the intercostal space, a sign known as Gray Turner's sign, indicative of hemorrhage in the retroperitoneal space. Incorrect Answers: Answer 1: ERCP is appropriate if obstruction is suspected, this patient's disease is likely alcohol mediated. Answer 3: This is an appropriate approach to a patient presenting with alcohol withdrawal. Answer 4: This patient has no evidence of cirrhosis or stigmata of portacaval anastomosis. Answer 5: PTC is appropriate in patients with biliary obstruction who fail ERCP.

A 60-year-old man presents to the emergency department with abdominal and flank pain on the left side. He states that it started 2 days ago and has been worsening. The patient has a past medical history of diabetes and hypertension and recently completed a course of oral antibiotics for diverticulitis. His temperature is 101°F (38.3°C), blood pressure is 161/97 mmHg, pulse is 107/min, respirations are 15/min, and oxygen saturation is 99% on room air. Physical exam does not reveal any abdominal or costovertebral angle tenderness. Flexion of the left leg against resistance elicits pain as does extension of the left hip. Which of the following is the most appropriate next step in management? Colonoscopy CT scan of the abdomen and pelvis Kidneys, ureters, and bladder radiograph Ultrasound Urinalysis

B, Appendicitis This patient is presenting with a recent infection (diverticulitis) followed by a fever, abdominal pain, flank pain, and a positive psoas sign which is concerning for a psoas abscess. The best diagnostic test for a psoas abscess is a CT scan. A psoas abscess occurs when an abscess forms in the psoas muscle due to its rich blood supply and proximity to lymphatic channels. Infection is typically caused by Staphylococcus aureus. Patients will present with flank pain, abdominal pain, hip pain, and a fever. They will have pain elicited when flexing the affected leg against resistance or by extending the affected leg. The diagnosis can be confirmed with a CT scan. Treatment involves both IV antibiotics as well as possible surgical drainage. Incorrect Answers: Answer 1: Colonoscopy would be indicated if there was a concern for colon cancer. Colon cancer would be suspected in an elderly patient with a microcytic anemia and guaiac-positive stools. Answer 3: Kidneys, ureters, and bladder radiograph would not offer sufficient detail to support the diagnosis of a psoas abscess. Similarly, it would be inferior to a CT scan without contrast for ruling out a kidney stone. Answer 4: Ultrasound could be used to assess for hydronephrosis if looking for a kidney stone or a soft tissue abscess; however, it would not be able to reliably diagnose a psoas abscess. Answer 5: Urinalysis could be indicated to confirm the diagnosis of a UTI which presents with urinary burning and frequency or to support the diagnosis of a kidney stone which would demonstrate hematuria. Bullet Summary: A CT scan is the diagnostic test of choice for a psoas abscess.

A 23-year-old man presents to the emergency department acutely intoxicated. The patient is not able to offer a coherent history but is actively vomiting. The vomit appears to have blood in it. He does state that he is having chest pain. The patient's vomiting improves with ondansetron. Initial laboratory values are ordered and are currently pending. His temperature is 99.2°F (37.3°C), blood pressure is 139/87 mmHg, pulse is 112/min, respirations are 16/min, and oxygen saturation is 95% on room air. A chest radiograph is performed as seen in Figure A. Which of the following is the most likely diagnosis? Aspiration pneumonia Boerhaave syndrome Mallory-Weiss tear Mediastinitis Pneumothorax

B, Boerhaave Syndrome This patient is presenting after profuse vomiting with chest pain and a radiograph demonstrating mediastinal free air which is concerning for a diagnosis of Boerhaave syndrome. Boerhaave syndrome is a transmural tear of the esophagus that occurs after repeat vomiting/wretching. Patients will complain of bloody vomitus and dysphagia/odynophagia as well as chest pain and subcutaneous emphysema. A chest radiograph will demonstrate free mediastinal air. The diagnosis can be supported with a gastrografin swallow or a CT scan. Patients with an esophageal tear should immediately be given broad-spectrum antibiotics and have a surgical repair of the lesion. Figure/Illustration A is a chest radiograph demonstrating mediastinal free air (red arrows). Incorrect Answers: Answer 1: Aspiration pneumonia would present with a fever and a linear consolidation in the lung in the setting of vomiting. It is too early in this patient's course of vomiting to have developed an aspiration pneumonia (rather, the patient would have an aspiration pneumonitis this early on) and the chest radiograph demonstrates mediastinal free air. Answer 3: Mallory-Weiss tear presents with bloody vomitus after repeated vomiting secondary to a submucosal tear in the esophagus. It is not a transmural tear and would not present with mediastinal free air on chest radiograph. Answer 4: Mediastinitis is a life-threatening complication where a patient presents with a fever, chest pain, unstable vitals, and mediastinal widening on chest radiograph typically after surgery in the area. Patients need immediate drainage/debridement and broad-spectrum antibiotics. Answer 5: Pneumothorax presents with pleuritic chest pain and hypoxia and on chest radiograph there would be lung markings that do not extend to the periphery. A small spontaneous pneumothorax can be managed with oxygen or a pigtail catheter. Bullet Summary: Boerhaave syndrome presents after profuse vomiting with chest pain, odynophagia, and mediastinal air on chest radiograph.

A 55-year-old obese man presents to the emergency department with abdominal pain. His pain started 2 days ago and has been worsening steadily. His only current medication is allopurinol. His temperature is 101.0°F (38.3°C), blood pressure is 157/98 mmHg, pulse is 120/min, respirations are 17/min, and oxygen saturation is 98% on room air. Physical exam is notable for left lower quadrant abdominal tenderness and rectal exam is unremarkable. A CT scan is performed demonstrating inflamed outpouchings of the colonic mucosa with a 4-cm abscess. Which of the following is appropriate treatment of this patient? Ciprofloxacin and gentamicin CT-guided percutaneous drainage Laparoscopy and drainage Piperacillin-tazobactam and vancomycin Supportive therapy and observation until symptoms abate

B, Diverticulitis This patient is presenting with diverticulitis with an abscess which must be treated with IV antibiotics and CT-guided percutaneous drainage. Diverticulitis occurs when outpouchings of the colonic mucosa became inflamed and infected. This causes the presentation of fever and left lower quadrant abdominal tenderness. The best initial diagnostic test for diverticulitis is a CT scan of the abdomen. For cases of uncomplicated diverticulitis, the patient can be treated with ciprofloxacin and metronidazole. Diverticulitis with an abscess requires more invasive therapy including percutaneous CT-guided drainage followed by IV antibiotics. Once the patient's clinical picture has been stabilized, they can be transitioned to oral antibiotics which include ciprofloxacin and metronidazole. Incorrect Answers: Answer 1: Ciprofloxacin and gentamicin is not the typical antibiotic regimen used for diverticulitis. Diverticulitis is typically treated with ciprofloxacin and metronidazole. An abscess > 3 cm requires drainage. Answer 3: Laparoscopy and drainage are not necessary to drain the abscess as a less invasive procedure such as CT-guided percutaneous drainage is sufficient. Answer 4: Piperacillin-tazobactam and vancomycin is broad coverage which could be used for diverticulitis and is insufficient as the abscess must first be drained. Piperacillin-tazobactam would be sufficient antibiotic coverage and the addition of vancomycin is unnecessary for diverticulitis with an abscess. Answer 5: Supportive therapy and observation until symptoms abate is the appropriate management of diverticulosis which presents with painless blood per rectum. Bullet Summary: Diverticulitis with an abscess must be treated with CT-guided percutaneous drainage and IV antibiotics.

A 71-year-old female presents to her primary care physician with a four-day history of left lower quadrant abdominal pain. The patient also complains of diarrhea and mild nausea. She continues to tolerate a diet, though reduced in quantity. On exam, her vitals are T 101F HR 86, BP 130/92, RR 15, and SaO2 100%. Abdominal exam is notable for left lower quadrant tenderness to palpation without rebound or guarding. Urine dipstick is normal and complete blood count shows a minor leukocytosis with a left shift. A screening colonoscopy from a year ago is shown in Figure A. What is the next best step in management? Immediate colonoscopy Trimethoprim-sulfamethoxazole and a liquid diet Abdominal CT with IV contrast CT angioplasty of mesenteric vessels Laparotomy and surgical management

B, Diverticulitis This woman with a history of diverticulosis presents with signs and symptoms of acute diverticulitis. The treatment of choice for a first episode of uncomplicated diverticulitis is bowel rest, antibiotics (trimethoprim-sulfamethoxazole), and a liquid diet. Twenty to twenty-five percent of individuals with diverticulosis will develop diverticulitis in their lifetime. Typical symptoms include left lower quadrant abdominal pain, nausea, dysuria, and diarrhea or constipation. Patients may present with peritonitis if there is an intraabdominal abscess. For a first episode of mild diverticulitis, treatment includes bowel rest, low roughage diet, and followup in 2-3 days. Despite evidence showing a lack of efficacy of broad spectrum antibiotics, most patients managed as outpatients are still prescribed broad spectrum antibiotics with regimens such as trimethoprim-sulfamethoxazole or fluoroquinolones + metronidazole. Wilkins et al. discuss the diagnosis and management of acute diverticulitis. They conclude that antibiotic therapy does not accelerate recovery or prevent complications or recurrence of diverticulitis. For patients with peritonitis or concern for complications such as abscess or fistula formation, inpatient management and imaging (CT with PO and IV contrast) is warranted. Additionally, as many as 30% of individuals hospitalized for diverticulitis will require surgical intervention, with laparoscopic surgery being associated with shortened length of hospital stay. Shabanzadeh et al. discuss the role of antibiotic therapy for uncomplicated diverticulitis. In their meta-analysis they found that there was no statistically significant difference between management with and without antibiotics. They conclude that as more studies elucidate the impact or lack thereof of antibiotic therapy in uncomplicated diverticulitis, clinical guidelines will have to be adjusted to fit the most up to date science. Figure A shows endoscopic view of the sigmoid colon with multiple diverticula in view, diagnostic for diverticulosis. Incorrect Answers: Answer 1: Immediate colonoscopy would not be appropriate due to the risk of rupture or complications. Follow up colonoscopy in 4-6 weeks after resolution is appropriate in complicated diverticulitis. Answer 3: The decision to obtain further imaging in diverticulitis is largely driven by the patient's presentation. In a patient with mild symptoms who is tolerating a diet, outpatient management is appropriate. Answer 4: CT angiography is the diagnostic tool of choice for mesenteric ischemia, not diverticulitis. Answer 5: Laparotomy and surgical management is reserved for patients with evidence of perforation or ischemia. Sigmoid colectomy for diverticulitis is the treatment of choice for complicated or recurrent diverticulitis after resolution of the acute episode.

A 72-year-old man presents to the emergency department when he discovered a large volume of blood in his stool. He states that he was going to the bathroom when he saw a large amount of bright red blood in the toilet bowl. He was surprised because he did not feel pain and felt it was a normal bowel movement. The patient has a past medical history of diabetes, obesity, hypertension, anxiety, fibromyalgia, diabetic nephropathy, and schizotypal personality disorder. His current medications include atorvastatin, lisinopril, metformin, insulin, clonazepam, gabapentin, sodium docusate, polyethylene glycol, fiber supplements, and ibuprofen. His temperature is 99.5°F (37.5°C), blood pressure is 132/84 mmHg, pulse is 80/min, respirations are 11/min, and oxygen saturation is 96% on room air. On physical exam, the patient's cardiac exam reveals a normal rate and rhythm, and his pulmonary exam is clear to auscultation bilaterally. Abdominal exam is notable for an obese abdomen without tenderness to palpation. Which of the following is an appropriate treatment for this patient's condition? Cautery of an arteriovenous malformation IV fluids and NPO NPO, ciprofloxacin, and metronidazole Surgical excision of poorly differentiated tissue Surgical resection of a blood vessel

B, Diverticulosis This patient is presenting with bloody stools in the absence of systemic symptoms suggesting a diagnosis of diverticulosis. The treatment of diverticulosis is supportive therapy (IV fluids and NPO). Diverticulosis typically occurs in patients with longstanding constipation. The increased abdominal pressure causes outpouching of the colonic mucosa forming diverticula which can bleed. Diverticula tend to develop at points of weakness where the vasa recta penetrate the circular muscle of the colon. Abnormal colonic motility and exaggerated segmentation contractions contribute to the pathophysiology. In cases of diverticulosis, symptoms resolve spontaneously and all that is needed is supportive therapy. Long-term management includes fiber supplements to treat underlying constipation and prevent future episodes/worsening of the condition. Incorrect Answers: Answer 1: Cautery of an arteriovenous malformation is an appropriate treatment for angiodysplasia which also presents with painless bright red blood per rectum. Epidemiologically, diverticulosis is a more likely diagnosis, in particular with a medical history of constipation. Answer 3: NPO, ciprofloxacin, and metronidazole is appropriate treatment of diverticulitis. Diverticulitis would present with systemic symptoms, left lower quadrant abdominal tenderness, and would not present with such a copious amount of blood in the stool. Answer 4: Surgical excision of poorly differentiated tissue is the management of colon cancer. Given this patient's sudden and voluminous bloody stool and lack of risk factors for colon cancer (aside from old age), diverticulosis is a more likely diagnosis. Answer 5: Surgical resection of a blood vessel is the appropriate management of hemorrhoid that is symptomatic/bleeds frequently. There was no mention of external hemorrhoids on physical exam, and such voluminous blood in the setting of constipation makes diverticulosis a more likely diagnosis. Bullet Summary: Diverticulosis is treated with supportive therapy (NPO and fluid support).

A 62-year-old man presents to the emergency department concerned about a large amount of blood in his recent bowel movement. He states he was at home when he noticed a large amount of red blood in his stool. He is not experiencing any pain and otherwise feels well. The patient has a past medical history of diabetes and obesity. His temperature is 98.9°F (37.2°C), blood pressure is 147/88 mmHg, pulse is 90/min, respirations are 13/min, and oxygen saturation is 98% on room air. Physical exam reveals a non-distressed man. His abdomen is non-tender, and he has normoactive bowel sounds. Stool guaiac test is positive for blood. The patient is started on IV fluids and kept nil per os. His next bowel movement 4 hours later appears grossly normal. Which of the following interventions will most likely reduce future complications in this patient? Ciprofloxacin and metronidazole Increase fiber and fluid intake Reduce red meat consumption Sigmoid colon resection Sitz baths

B, Diverticulosis This patient is presenting with symptoms suggestive of diverticulosis for which the best long-term management is to increase his fiber and fluid intake. Diverticulosis typically presents with bloody bowel movements in the setting of a patient who suffers from constipation. If the diverticulae become infected, the disease can progress to diverticulitis, which presents with abdominal pain, fever, and guaiac-positive stools. Increased fiber and fluid intake can reduce constipation and improve outcomes in diverticulosis. Incorrect Answers: Answer 1: Ciprofloxacin and metronidazole are appropriate antibiotics for diverticulitis; however, this patient has no signs of infection/diverticulitis. Answer 3: Reducing red meat consumption could reduce the risk of diverticulitis if the red meat is replaced with fiber and fluids. Red meat consumption is more readily associated with colon cancer and poor cardiovascular health. Answer 4: Sigmoid colon resection could be indicated in patients who suffer from repeat episodes of diverticulitis to prevent future episodes from occurring. Answer 5: Sitz baths are an appropriate part of the management of external hemorrhoids as they reduce the associated pain/symptoms. Bullet Summary: Increasing fiber and fluid intake in diverticulosis can reduce future complications.

A 68-year-old male smoker with a history of peripheral vascular disease and a below the knee amputation, presents to the emergency room with excruciating abdominal pain and vomiting for the last 6 hours. Vital signs are T 101.0 F HR 136 BP 150/96 RR 18 Sat 93% on room air. Abdominal exam shows a non-distended, exquisitely tender abdomen which the patient refuses to allow you to examine. While assessing peripheral pulses you note that they are irregular. A digital rectal exam is Guiac positive. Laboratory values are notable for a leukocytosis and a lactic acid of 6.8 mmol/L. An ECG is shown in Figure A. What is the next best step in diagnosis? Diagnostic peritoneal lavage CT Angiography Technetium-99 tagged RBC scintigraphy Upper Endoscopy Upper GI with follow through

B, Mesenteric Ischemia This patient with a history of atherosclerotic disease, presents with signs and symptoms of acute mesenteric ischemia (AMI), likely due to arterial thrombosis secondary to new onset atrial fibrillation. The best step in diagnosis for mesenteric ischemia is CT angiography. Chronic mesenteric ischemia (CMI) occurs in the setting of atherosclerotic disease, with risk factors including smoking, diabetes, and dyslipidemia. Patients with CMI may present with abdominal pain exacerbated or provoked by eating and may develop food aversion, which can result in substantial weight loss and malnutrition. AMI presents upon sudden occlusion of mesenteric vessels, commonly the celiac axis or the superior mesenteric artery. Risk factors for AMI include previous CMI, arterial catheterization proximal to the takeoff of mesenteric vessels, new thrombogenic arrhythmias such as atrial fibrillation or atrial flutter. Patients present with pain out of proportion of exam. Prompt surgical intervention is key to outcomes, with revascularization being the goal. In prolonged ischemia, necrosis may develop and can present radiographically as pneumatosis intestinalis with portal venous gas (Illustration A). Hirsch et al. discusses the natural history or peripheral arterial disease (PAD), including chronic and acute mesenteric ischemia. The greatest identified risk factors for developing PAD is notably smoking and diabetes. Postprandial abdominal pain that is reproduced or provoked by eating and is associated with weight loss should provoke consideration for chronic mesenteric ischemia. Of note, patients who develop severe abdominal pain after arterial interventions in which catheters pass the visceral aorta, or in whom have arrhythmias such as atrial fibrillation, or recent myocardial infarction, should be suspected of acute mesenteric ischemia. Wyers discusses the diagnosis and management of acute mesenteric ischemia. The author concludes that the use of biphasic CT angiography has surpassed conventional angiography as the diagnostic method of choice because of its ability to confirm vascular occlusion, but also because of its capacity to evaluate for secondary signs of mesenteric ischemia such as pneumatosis intestinalis (Illustration A), along with other possible etiologies of the abdominal pain. Figure A shows an abnormal EKG with atrial fibrillation (irregularly irregular QRS interval spacing with absence of P-Waves). Illustration A shows a coronal reconstruction of an abdominal CT scan with pneumatosis intestinalis and portal venous gas, suggestive of bowel ischemia and gangrene. Incorrect Answers Answer 1: Diagnostic peritoneal lavage is a largely historical procedure, that has been replaced almost completely by the bedside ultrasound, and readily available CT imaging. It has no role in the diagnosis of acute mesenteric ischemia. Answer 3: Technetium-99 tagged RBC scintigraphy is sometimes used to help evaluate for lower GI bleeding of an unknown source. Answer 4: Upper endoscopy is used to evaluate intraluminal pathology of the foregut. It has very limited role in the diagnosis of acute mesenteric ischemia. Answer 5: An upper GI radiograph series with follow through is useful in diagnosing ileus or delayed gastric emptying. Patients with suspected mesenteric ischemia should remain strictly NPO.

A 76-year-old female is brought to the emergency department after being found unresponsive in her room at her nursing facility. Past medical history is significant for Alzheimer's disease, hypertension, and diabetes. Surgical history is notable for an open cholecystectomy at age 38 and multiple cesarean sections. On arrival, she is non-responsive but breathing by herself, and her vital signs are T 102.9 F, HR 123 bpm, BP 95/64, RR 26/min, and SaO2 97% on 6L nasal cannula. On physical exam the patient has marked abdominal distension and is tympanic to percussion. Laboratory studies are notable for a lactic acidosis. An upright abdominal radiograph and CT abdomen/pelvis with contrast are shown in Figures A and B respectively. She is started on IV fluids and a nasogastric tube is placed to suction which returns green bilious fluid. Repeat vitals 1 hour later are T 101F, HR 140 bpm, BP 75/44, RR 30/min, and SaO2 is 100% on the ventilator after she is intubated for airway concerns. What is the next best step in management? Therapy with levofloxacin and metronidazole Immediate laparotomy and surgical management Continue IV fluid hydration, nasogastric suction, NPO Pneumatic enema Sigmoidoscopy, attempted derotation and rectal tube placement

B, Small Bowel Obstruction This patient with an extensive surgical history presents with a small bowel obstruction and likely small bowel ischemia. After worsening on conservative therapy, immediate surgical management is required, with laparotomy being the approach of choice. Small bowel obstructions account for around 15% of all surgical admissions in the United States. Adhesive disease, typically secondary to previous surgery, accounts for around 75% of all small bowel obstructions, with malignancy and hernias accounting for another 16% combined. Conservative management with nasogastric suction, IV fluid hydration, and bowel rest is an appropriate first strategy for patients who present without evidence of peritonitis and who are hemodynamically stable. Patients who fail conservative management or who present with more advanced disease require immediate surgical management. Jackson et al. discuss the diagnosis and management of intestinal obstructions in adults. They conclude that in the hemodynamically stable patient, IV fluid hydration, nasogastric intestinal decompression, and bowel rest is an appropriate first strategy to attempt to resolve the obstruction. However, hemodynamically unstable patients, patients with perforations or ischemic gut, and patients who fail conservative management require surgical intervention. O'Connor et al. discuss the growing role of laparoscopic surgical approaches for acute small bowel obstruction. They conclude that with the limited literature available, laparoscopic lysis of adhesions is an appropriate approach for the treatment of adhesive small bowel disease. In this study, ~30% of laparoscopic approaches were converted to midline laparotomies. Laparoscopy was associated with reduced morbidity and length of hospital stay. Further studies that randomize patients to open vs. laparoscopic surgical approaches are required to further elucidate the outcomes of these two approaches. Figure A shows a partially upright abdominal radiograph with dilated loops of small bowel and air fluid levels, consistent with a small bowel obstruction. Figure B shows an axial reconstruction of an abdominal CT scan, showing dilated loops of small bowel. Red arrow is pointing at an air fluid level. Illustration A shows an abdominal radiograph with significantly dilated sigmoid colon, with a characteristic coffee bean sign highly suggestive of sigmoid volvulus and obstruction. Incorrect Answers Answer 1: Broad spectrum antibiotics is not an appropriate strategy for management of small bowel obstruction. Answer 3: Continuing conservative management with IV fluid hydration, NG suction, and NPO is appropriate in hemodynamically stable patients, but this patient presents with worsening shock. Answer 4: Pneumatic enema is the procedure of choice for patients with intussusception, as it is both diagnostic and therapeutic. It has no role in the management of adhesive small bowel obstructions. Answer 5: Sigmoidoscopy with attempted derotation is a reasonable approach to patients presenting with sigmoid volvulus. These patients can present similarly to those with a small bowl obstruction; however, this patient's radiograph is suggested of a small bowel source. See Illustration A

A 5 day old neonate presents at night to the emergency room with "one day of green vomiting and a larger belly than usual" according to mom. The mother explains that the patient passed stool for only two days after birth and had been feeding well until this morning when the patient vomited after feeding and continued to vomit sporadically throughout the day. Newborn screening was normal for this patient, there were no pregnancy complications, the patient has been breastfeeding consistently with only some regurgitation, and the patient has no sick contacts. The abdomen is distended on physical exam without palpable masses, and there is no stool in the rectal vault. Upper gastrointestinal series film can be appreciated in Figure A. What is the most likely diagnosis of this patient? Duodenal Atresia Mid-gut Volvulus Pyloric Stenosis Biliary Atresia Meconium Ileus

B, Volvulus A 5-day old infant presenting with green colored emesis, distention, and dilated loops of bowel on radiograph most likely has neonatal malrotation with volvulus. Malrotation with volvulus occurs due to an incomplete malrotation of the intestines during the 7-12th weeks of development. Patients will feed well until 3-7 days of life when bilious vomiting, abdominal distention, and rapid decompensation may occur. Complications of volvulus include intestinal ischemia, infection, necrosis, perforation of bowel, and ultimately shock. Kimura et al. discuss the importance of rapid diagnosis of small bowel obstruction following bilious vomiting and the importance of rapid treatment to prevent major complications. Following observation of bilious vomiting and abdominal distention, physicians must place a naso-gastric or oro-gastric tube for decompression followed by plain abdominal radiographs. If dilated loops of bowel and/or air fluid levels are observed intestinal obstruction should be suspected. Luang et al. discuss the treatment and cardinal signs of neonatal obstruction, which include maternal polyhydramnios, bilious emesis, failure to pass meconium first day, and abdominal distention. Radiographs aid in diagnosis but shouldn't delay treatment if suspected and treatment includes, NPO, fluid resuscitated, intravenous broad-spectrum antibiotics, and immediate operative exploration. Definitive treatment is the ladd procedure which detorses the bowel. Figure A demonstrates barium flowing in a corkscrew pattern in an upper gastrointestinal series. This is a common finding of midgut volvulus. Incorrect answers: Answer 1: Signs of duodenal atresia occur immediately after birth with bilious emesis and no abdominal distention. Answer 3: Pyloric stenosis does not present with bilious emesis or distention, occurs 3-5 weeks after birth, and an olive shaped mass would be palpated on physical exam. Answer 4: Biliary atresia does not present with distention, bilious vomiting, or these radiographic findings. Answer 5: Meconium ileus presents in babies with cystic fibrosis and, whot typically fail to pass stool on the first day of life.

A 19-year-old college student presents to the emergency department with excruciating retrosternal chest pain that radiates to the back. She describes the pain as sharp and worsens with swallowing. She reports that her symptoms began after vomiting 1 hour ago. Medical history is significant for anorexia nervosa. Her temperature is 100.4°F (38°C), blood pressure is 135/90 mmHg, pulse is 105/min, and respirations are 20/min. On physical exam, there is crepitus upon chest palpation. A barium esophagram is performed and shows leakage of water-soluble contrast from the lower thoracic esophagus.

Boerhaave Syndrome (snapshot)

A 19-year-old woman presents to the emergency department with severe retrosternal chest pain. Her symptoms began immediately after vomiting. Medical history is significant for bulimia nervosa. Physical examination is notable for crepitus upon chest wall palpation. A barium swallow study is shown.

Boerhaave Syndrome (snapshot)

A 44-year-old obese woman presents with abdominal pain. She says the pain started while she was having lunch at a fast-food restaurant with her children. The pain began shortly after eating and has persisted for 6 hours. She has vomited once. Her vital signs are as follows: HR 88, BP 110/70 mmHg, T 38.5°C (101.3°F). On physical exam, she is tender to palpation in the right upper quadrant of her abdomen. Her skin appears normal. Her liver function tests, amylase, and lipase levels are normal. A right upper quadrant abdominal ultrasound is challenged by her body habitus and is not able to visualize any gallstones. Which of the following is the most likely cause of her presentation? Acalculous cholecystitis Cancer of the biliary tree Gallstone disease Pancreatic inflammation Cholangitis

C, Acute Cholecystitis This patient has cholecystitis most likely due to gallstone disease, even though no stones were seen on the ultrasound. Cholecystitis classically presents with RUQ tenderness, fever, and a positive Murphy's sign (inspiratory arrest during deep palpation of the RUQ). Following obstruction, bacteria can proliferate in the stagnant fluid. Commonly associated bacteria include: E. coli (most common cause); Enterobacter cloacae; B. fragilis; Clostridium spp. In more severe cases, bacteria can invade the gallbladder wall, and this can lead to gangrenous necrosis and perforation. Bellows et al. discuss the pathophysiology of acute cholecystitis. Acute cholecystitis is caused by complete obstruction of the cystic duct, usually due to an impacted stone. Bile stasis occurs, and the gallbladder may become inflamed. In contrast to biliary colic, which also arises from stones occluding the gallbladder outlet, in acute cholecystitis the obstruction and pain persists for at least 4-6 hours. Negative ultrasound can be seen in gallstone/calculous cholecysitis. Although specificity is high, in the 95-99% range, sensitivity is not as high, with reported values in the literature ranging between 84% to 99%. These false negatives can especially be seen in obese patients, or patients who recently passed the stone. Kimura et al. review the causes of cholecystitis other than gallstones (thus known as acalculous cholecystitis). Ischemia of the gallbladder, motility disorders, direct injury, infection (particularly with protozoal or parasitic organisms), collagen diseases, or allergic conditions can lead to cholecystitis without gallstones. All together, gallstones account for 90-95% of all cases. Illustration A depicts a normal versus an inflamed gallbladder. The inflamed gallbladder shows stones and a thickened, inflamed wall. Illustration B demonstrates the ultrasound findings of acute cholecystitis, showing a thickened gallbladder wall and free pericholecystic fluid. Incorrect Answers: Answer 1: Acalculous cholecystitis classically occurs in very sick ICU patients. It is a relatively rare cause of cholecystitis, especially in otherwise healthy individuals. Acalculous cholecystitis can be related to a variety of predisposing defects (as listed above), but is by definition not caused by gallstones. However, as in this question, acute cholecystitis may be caused by a gallstone that is not visualized on radiological examination, either because the stone has passed or because the radiologic study was not diagnostic. But when this occurs, the correct diagnosis is still gallstone cholecystitis, not acalculous cholecystitis. Answer 2: Cholangiocarcinoma is an uncommon malignancy of the bile ducts, which usually presents with jaundice and more severe illness. Answer 4: Pancreatic inflammation is suggestive of pancreatitis, but this is ruled out with normal amylase and lipase. Answer 5: Cholangitis is a significant infection of the biliary tree. In cholangitis, patients are very sick and may present with hypotension and altered mental status.

A 48-year-old female presents to the emergency department with fevers, and worsening abdominal pain for 24 hours. Exam demonstrates an overweight female in distress. Abdominal exam is notable for tenderness to palpation in the right upper quadrant and a positive ultrasonic (US) murphy's sign. Right upper quadrant ultrasound is shown in Figure A. Vital signs are as follows: T 102.1 F HR 84 BP 135/92 RR 14 O2 Sat 97%. Lab studies demonstrate leukocytosis. What's the next best step in management? IV fluid hydration, analgesics, NPO, Interval cholecystectomy after 7 days antibiotic therapy IV fluid hydration, analgesics, NPO, Open cholecystectomy IV fluid hydration, analgesics, NPO, Laparoscopic cholecystectomy IV fluid hydration, analgesics, NPO, Hida Scan IV fluid hydration, analgesics, NPO, Endoscopic retrograde cholangiopancreatography (ERCP)

C, Acute Cholecystitis This patients presents with acute calculous cholecystitis. Definitive surgical management in the initial hospitalization should occur, with laparoscopic cholecystectomy being the operation of choice. Acute calculus cholecystitis is caused by obstruction of the gallbladder neck by a gallstone. This obstruction leads to increased pressure and distention of the gallbladder. Over time, edema, release of inflammatory mediators, and bacterial extravasation into the parenchyma lead to inflammation. The management of acute cholecystitis involves IV hydration, analgesia for pain, nil per os, and surgical excision. Surgical management should occur during the initial hospitalization, and within 72 hours of symptomatic onset. If surgery isn't possible within this initial period, an interval cholecystectomy after antibiotic therapy is preferred. Abraham et. al. discuss the different management strategies for acute cholecystitis. They conclude that laparoscopic cholecystectomy remains the surgery of choice, as it has an improved recovery time with a shorter length of hospital stay. For patients with symptomatic cholecystitis, who are too ill for surgery, decompression with percutaneous cholecystostomy is a viable strategy. Yang et. al. reviewed the risk factors for conversion of laparoscopic cholecystectomy to open cholecystectomy. They found that prior abdominal surgery, concomitant diabetes, age greater than 65, male gender, and thickened gallbladder wall were independent risk factors for surgical conversion. Figure A shows a right upper quadrant ultrasound demonstrating gallbladder thickening and gallstones, consistent with acute cholecystitis. Illustration A shows an ERCP with endoscopic stone retrieval after sphincterotomy. Incorrect Answers Answer 1: Interval cholecystectomy isn't recommended, as this patient is within the 72-hour window from symptom onset. Answer 2: Open cholecystectomy is no required, as a laparoscopic approach is preferred. Answer 4: a HIDA Scan is a diagnostic tool for diagnosis of cholelithiasis, this patient has sufficient diagnostic data to conclude acute cholecystitis. Answer 5: ERCP is both diagnostic and therapeutic for choledocholithiasis (See Illustration A). There is no evidence of common duct stones in this patient, and if there is concern, an intraoperative cholangiogram is the procedure of choice.

A 45-year-old homeless man presents to the emergency department acutely intoxicated. He has a history of belligerence and substance abuse and was found today in a local park outdoors in Arizona. His temperature is 97.7°F (36.5°C), blood pressure is 100/66 mmHg, pulse is 130/min, respirations are 26/min, and oxygen saturation is 98% on room air. The patient is responsive to sternal rub and cries out in pain with palpation of his epigastric region. Lower extremity petechiae are noted. Laboratory studies are ordered as seen below. Hemoglobin: 10 g/dL Hematocrit: 31% Leukocyte count: 6,520/mm^3 with normal differential Platelet count: 51,000/mm^3 Serum: Na+: 130 mEq/L Cl-: 101 mEq/L K+: 3.3 mEq/L HCO3-: 25 mEq/L BUN: 20 mg/dL Glucose: 66 mg/dL Creatinine: 1.7 mg/dL Ca2+: 10.2 mg/dL INR: 2.5 A peripheral smear is ordered as seen in Figure A. The patient is given 2 liters of fluids, and his pulse is subsequently 80/min with a blood pressure of 125/81 mmHg. Which of the following is the most likely diagnosis? Cirrhosis Immune thrombocytopenic purpura Pancreatitis Rattlesnake bite Septic shock

C, Acute Pancreatitis This alcoholic patient is presenting with epigastric pain, a low platelet count (with petechiae), an elevated INR, and schistocytes on peripheral smear which is concerning for disseminated intravascular coagulation (DIC) caused by pancreatitis. Pancreatitis occurs when there is inflammation/injury to the pancreas from gallstones, alcohol ingestion, or infection. It can present with epigastric pain, nausea, vomiting, Grey-Turner sign (flank ecchymosis), Cullen sign (umbilical ecchymosis), and an elevated lipase level. The management of pancreatitis is to keep the patient nil per os (NPO), administer IV fluids, and control the pain. A severe complication of pancreatitis is disseminated intravascular coagulation (DIC). Pancreatic enzymes can pathologically activate the coagulation cascade leading to a depletion of coagulation factors (an elevated PT/PTT), depletion of platelets, and schistocytes from the shearing of the red blood cells from the microvasculature thrombi created from activation of the coagulation cascade. Figure/Illustration A is a peripheral smear demonstrating schistocytes (blue arrows) or helmet cells which are commonly seen in DIC. Incorrect Answers: Answer 1: Cirrhosis would present with jaundice, scleral icterus, abdominal ascites, esophageal varices, caput medusae, and hemorrhoids. While liver failure may impair the synthesis of coagulation factors and increase this patient's INR, it would not lead to a decreased platelet count in the setting of schistocytes, which is seen in DIC. Answer 2: Immune thrombocytopenic purpura occurs secondary to the autoimmune destruction of platelets. Patients will present with petechiae and a low platelet count. Treatment involves observation in stable/non-bleeding patients with a platelet count > 50,000 cells/uL, or IVIG/steroids in more severe cases. Answer 4: Rattlesnake bite is a classic but rare cause of DIC. A rattlesnake bite would present with 2 parallel bite marks that are actively oozing with erythema, edema, tenderness, and swelling of the bitten extremity. Rattlesnake venom can activate the coagulation cascade leading to DIC. Answer 5: Septic shock can cause DIC and would present with a fever, hypotension, and tachycardia. This patient may have an occult infection; however, his response to fluids with a normal heart rate and a normal blood pressure by definition makes this not septic shock (though it could be sepsis). Sepsis is a possible cause of DIC, though no localizing infectious symptoms are found in this patient. Bullet Summary: Pancreatitis can lead to pathologic activation of the coagulation cascade causing disseminated intravascular coagulation.

An 8-year old boy with no past medical history presents to the emergency room with 24-hours of severe abdominal pain, nausea, vomiting, and non-bloody diarrhea. His mom states that he has barely eaten in the past 24 hours and has been clutching his abdomen, first near his belly button and now near his right hip. His temperature is 101.4°F (38.5°C), blood pressure is 101/63 mmHg, pulse is 100/min, and respirations are 22/min. On physical exam, the patient is lying very still. There is abdominal tenderness and rigidity upon palpation of the right lower quadrant. What is the most likely cause of this patient's clinical presentation? Granulomatous inflammation of the appendix Diverticulum in the terminal ileum Appendiceal lymphoid hyperplasia Structural abnormality of the appendix Twisting of the spermatic cord

C, Appendicitis This child's right lower quadrant pain, anorexia, nausea and vomiting, and fever is highly concerning for appendicitis. Appendicitis is commonly caused by appendiceal lymphoid hyperplasia in children. The abdominal pain in appendicitis classically presents as periumbilical pain that migrates to the right lower quadrant within the first 24 hours. The most common cause of appendicitis in children is lymphoid hyperplasia. Lymphoid follicles reside in the epithelium that lines the appendix and may increase in size following a viral infection or immunization. Growth of these follicles can obstruct the lumen of the appendix, leading to subsequent bacterial overgrowth within the appendix accompanied by inflammation. The increase in pressure within the appendix impairs blood flow and results in ischemia and possible perforation. Treatment is typically emergent abdominal surgery. Incorrect Answers: Answer 1: Granulomatous inflammation of the appendix can be observed in patients with Crohn's disease. However, this is a less common cause of appendicitis. Answer 2: A diverticulum of the terminal ileum, known as a Meckel's diverticulum, is an outpouching of the small intestine caused by the persistence of the omphalomesenteric duct. While a Meckel's diverticulum can cause abdominal pain that may mimic appendicitis, it most commonly presents earlier in childhood and it is most often clinically silent. The classic symptomatic presentation is painless bleeding, which is due to ectopic gastric mucosa within the diverticulum. Answer 4: A structural abnormality such as a bend or twist in the appendix can lead to obstruction of the appendiceal lumen and result in appendicitis. However, this is not as common a cause as lymphoid hyperplasia. Answer 5: In testicular torsion, twisting of the spermatic cord decreases blood flow to the testes. While patients with testicular torsion can present with abdominal pain, nausea, and vomiting, they would also have scrotal or testicular symptoms and signs on exam (tender, swollen, slightly elevated testicle, and often with absent cremasteric reflex). Bullet Summary: Appendicitis often presents with periumbilical pain that migrates to the right lower quadrant, accompanied by nausea and vomiting. The most common etiology in childhood is lymphoid hyperplasia secondary to viral infection or immunization.

A 45-year-old man is brought to the emergency department by police. He was found passed out in a store. The patient is dressed in soiled clothing that smells of urine and his pants are soaked in vomit. The patient has a past medical history of IV drug use, alcohol use, and multiple fractures secondary to scurvy. He is not currently taking any medications. His temperature is 99.5°F (37.5°C), blood pressure is 90/63 mmHg, pulse is 130/min, respirations are 15/min, and oxygen saturation is 95% on room air. The patient is started on IV fluids, thiamine, and dextrose. The patient becomes arousable at this point and begins vomiting into a basin. On physical exam, the patient appears poorly kempt and uncomfortable. His epigastric region is tender upon palpation. Cardiac exam reveals a normal rate and rhythm. Pulmonary exam is clear to auscultation bilaterally. A CT scan of the abdomen is ordered as seen in Figure A. Laboratory values are ordered as seen below: Serum: Na+: 139 mEq/L Cl-: 102 mEq/L K+: 4.0 mEq/L HCO3-: 26 mEq/L BUN: 20 mg/dL Glucose: 73 mg/dL Creatinine: 1.1 mg/dL Ca2+: 10.2 mg/dL Lipase: 295 U/L AST: 57 U/L ALT: 39 U/L Hemoglobin: 9 g/dL Hematocrit: 30% Mean corpuscular volume: 120 µm^3 Leukocyte count: 8,500/mm^3 with normal differential Platelet count: 199,000/mm^3 The patient continues to vomit and is observed in the emergency department. Several hours later his temperature is 99.5°F (37.5°C), blood pressure is 110/72 mmHg, pulse is 97/min, respirations are 15/min, and oxygen saturation is 95% on room air. Physical exam reveals a cardiac and pulmonary exam that are within normal limits. HEENT exam reveals suprasternal crepitus, an oropharynx that is blood-stained, and no signs of lymphadenopathy. Inspection of the patient's lower extremities is within normal limits. Which of the following is the best next step in management? Ondansetron Injection of epinephrine Gastrografin swallow Folate and vitamin B12 Supportive therapy

C, Boerhaave Syndrome This patient is presenting with profuse vomiting followed by suprasternal crepitus and blood-stained vomit suggesting a diagnosis of Boerhaave syndrome (esophageal rupture). Esophageal rupture is a surgical emergency that must first be diagnosed with a gastrografin swallow. Alcohol abuse can predispose patients to pancreatitis, megaloblastic anemia, and profuse vomiting. Intense vomiting can predispose patients to an esophageal mucosal tear (Mallory-Weiss syndrome) or an esophageal rupture (Boerhaave syndrome). Boerhaave syndrome presents with suprasternal/clavicular crepitus and bloody vomitus. The best initial step in management of an esophageal rupture is a gastrografin swallow to confirm the diagnosis followed by emergency surgery. Endoscopy is recommended for patients who are not surgical candidates. Figure A is a CT scan demonstrating edema of the pancreas suggesting a diagnosis of acute pancreatitis. Incorrect Answers: Answer 1: Ondansetron is an anti-emetic that could be used to control this patient's vomiting. This would be appropriate if this patient's only symptom were vomiting, or if he only had a Mallory-Weiss tear. Answer 2: Injection of epinephrine is the appropriate management of a Mallory-Weiss tear that does not stop bleeding in response to supportive therapy. Answer 4: Folate and vitamin B12 would treat this patient's megaloblastic anemia; however, it misses this patient's more acute and life-threatening diagnosis of esophageal rupture. Answer 5: Supportive therapy is the appropriate management of a Mallory-Weiss tear or pancreatitis. Though this patient has pancreatitis, it would not be appropriate management in the setting of esophageal rupture, which is a surgical emergency. Bullet Summary: Esophageal rupture should be managed with a gastrografin swallow followed by emergency surgery.

A 61-year-old man presents to his primary care provider complaining of abdominal pain and constipation. He reports a 4-day history of steady right lower quadrant pain. He has had one small bowel movement in 4 days. Normally he has a bowel movement once a day. His medical history is notable for poorly controlled hypertension and hyperlipidemia. He takes enalapril, hydrochlorothiazide, aspirin, and atorvastatin. He has a 40 pack-year smoking history and drinks 3-4 beers per day. His diet consists primarily of fast food. His temperature is 101.8°F (38.8°C), blood pressure is 160/95 mmHg, pulse is 90/min, and respirations are 16/min. A review of the patient's medical record reveals colonoscopy results from 1 year ago. Relevant findings included multiple small, pedunculated polyps which were removed, multiple colonic mucosal outpouchings, and no other masses. This patient's condition is most strongly associated with which of the following disorders? Alpha-1-antitrypsin deficiency Autosomal recessive polycystic kidney disorder Ehlers-Danlos syndrome Goodpasture syndrome Menetrier disease

C, Diverticular Disease The patient in this vignette presents with abdominal pain, fever, constipation, and previous imaging findings suggestive of underlying diverticulosis leading to acute diverticulitis. Diverticular disorders such as diverticulosis and diverticulitis are associated with collagen disorders such as Ehlers-Danlos syndrome. Diverticular disorders include diverticulosis and diverticulitis. Diverticulosis refers to the presence of multiple acquired diverticula while diverticulitis refers to infection and macroperforation of existing diverticula. The prevalence of diverticular diseases is quite high, but most patients are asymptomatic. Although the majority of colonic diverticula are left-sided, one-quarter are right-sided. Individuals with collagen disorders such as Ehlers-Danlos syndrome or Marfan syndrome are at an increased risk of developing diverticular diseases. Incorrect Answers: Answer 1: Alpha-1-antitrypsin deficiency (A1AT) is an autosomal codominant condition that leads to excess elastase activity. Clinically, this leads to panacinar emphysema and cirrhosis. A1AT is not strongly associated with diverticular disease. Answer 2: Autosomal recessive polycystic kidney disorder (ARPCKD) is a rare congenital disorder characterized by fibrocystic renal disease and end-stage renal disease by age 15. Most patients do not live to late adulthood. Diverticular disease is not strongly associated with ARPCKD; however, it is associated with the more common and less severe form of cystic kidney disease, autosomal dominant polycystic kidney disorder (ADPCKD). Answer 4: Goodpasture syndrome is an autoimmune disorder involving an auto-antibody against collagen type IV in the pulmonary and glomerular capillaries. This results in permanent and severe lung and kidney damage and often death. Answer 5: Menetrier disease is a rare pre-malignant stomach disorder characterized by hypertrophic gastropathy due to hyperplasia of mucus-producing cells. It is not strongly associated with diverticular disease. Bullet Summary: Patients with collagen disorders such as Marfan syndrome and Ehlers-Danlos syndrome are at an increased risk of developing diverticular disorders.

A 65-year-old man with a history of constipation is admitted to your floor with left lower quadrant pain. The patient states that he has been previously admitted with similar pain. He denies any medical history, but states that he does not like to eat vegetables. His wife states that he has had increasing pain over the last few days and that this time, the pain is more intense and that the patient has had higher fevers of up to 102°F (38.9°C). On exam, the patient's vitals are HR 102, T 101.9°F (38.8°C), BP 140/60, and RR 14. The patient has tenderness to palpation along with rebounding and guarding. Figure A shows a CT scan of the patient's abdomen and pelvis. What is the definitive treatment for this patient? Antibiotics and surveillance Discharge home Emergent operative treatment Percutaneous drainage Fluids

C, Diverticulitis An elderly patient with left lower quadrant pain in the setting of poor fiber intake and similar prior admissions is most likely to be suffering from diverticulitis. In this case, abdominal guarding and rebounding along with CT evidence of perforation suggest it to be perforated diverticulitis, which should be treated operatively. Diverticulitis is one of the most common causes of left lower quadrant pain in elderly patients. It increases in incidence with age and decreased fiber intake and is more common among Caucasian patients. Overgrowth of bacteria in the mucosal outpouchings of the sigmoid colon can cause inflammation, converting diverticulosis to diverticulitis, which can typically be treated with a course of antibiotics. Diverticulitis can be complicated by abscess, fistula, obstruction, bleeding, or perforation requiring additional, often operative, treatment. Wilkins et al. discuss the management of diverticulitis in the general population. They state that isolated left lower quadrant tenderness in elderly patients has a 10 fold increased likelihood of being caused by diverticulitis. Furthermore, they state that though uncomplicated diverticulitis can be treated on an outpatient basis, if the patient cannot tolerate PO intake or has any signs of peritonitis, the patient should be admitted for further testing. Khalil et al. discuss the management of perforated diverticulitis in patients. They state feculent peritonitis caused by perforated diverticulitis has a mortality rate ranging from 14% to 43%. Perforation should be treated emergently with a bowel resection and an end-to-end anastomosis or resection with a reversible stoma. Figure A shows an axial CT slice with a perforation in the sigmoid colon, likely in the setting of diverticulitis. Illustration A is a good example of complicated diverticulitis presenting with abscess formation. Incorrect Answers: Answer 1 and 5: Though this patient should receive antibiotics, surveillance and fluids alone are not the next best options for this patients since he has a perforated abdomen that can result in multiple, potentially lethal complications such as abscess formation, sepsis, and shock. Answer 2: The patient should not be discharged home because he is clearly sick and requires operative treatment. Answer 4: Percutaneous drainage may be appropriate if this patient presented with an accessible abscess, but this patient does not present with a drainable collection.

A 70-year-old man with a history of chronic constipation presents to the emergency department with a two-day history of left lower quadrant abdominal pain. He is found to have a temperature of 100.8F, BP 140/90, HR 85, and RR 16. On physical examination, he is tender to light palpation in the left lower quadrant and exhibits voluntary guarding. Rectal examination reveals heme-positive stool. Laboratory values are unremarkable except for a WBC count of 12,500 with a left shift. Which of the following tests would be most useful in the diagnosis of this patient's disease? Abdominal x-ray Lipase Abdominal CT Left lower quadrant ultrasound Emergent colonoscopy

C, Diverticulitis This patient most likely has acute diverticulitis, which is inflammation of colonic diverticula. CT scan is the modality of choice for the diagnosis of diverticulitis, with findings including pericolic fat stranding and bowel wall thickening. Diverticulitis is the most common complication of diverticular disease. Diverticula are outpouchings of the colonic mucosa through weak points in the bowel wall. They are present in 35-50% of the general population, with increased prevalence in the elderly and those with chronic constipation. Diverticulitis occurs when a diverticulum becomes obstructed, leading to inflammation, ischemia, and infection. This may progress to abscess formation or perforation with generalized peritonitis. Diverticulitis typically presents with left lower quadrant abdominal pain, fever, elevated white blood cell count with a left shift, and guaiac-positive stool. Though diverticula occur more often on the left side of the colon, those that rupture tend to occur on the right side. Wilkins et al. review the diagnosis and management of acute diverticulitis. They suggest that hospitalization should be considered if patients have signs of peritonitis or if there is suspicion for complicated diverticulitis. Of note, they write that 15 to 30% of patients admitted with acute diverticulitis will require surgical intervention. Longstreth et al. investigate the diagnoses of inflammatory bowel syndrome (IBS) and diverticulitis. They report evidence of misattribution of IBS pain to diverticulitis, especially in outpatients who were not worked up with a CT scan. This results in unnecessary antibiotic use. Illustration A shows a CT scan of a patient with acute diverticulitis. Note the bowel wall thickening and pericolic fat stranding inside the white circle. Incorrect Answers: Answer 1: Abdominal x-ray may reveal an ileus in the setting of diverticulitis, but it is not sensitive for the diagnosis of diverticulitis. Answer 2: Lipase levels are sensitive for the diagnosis of pancreatitis, not diverticulitis. Answer 4: Left lower quadrant ultrasound is not as sensitive as CT scan for the diagnosis of diverticulitis. Answer 5: Colonoscopy should be avoided in suspected cases of acute diverticulitis due to the risk of perforation.

A 75-year-old male is hospitalized for bloody diarrhea and abdominal pain after meals. Endoscopic work-up and CT scan lead the attending physician to diagnose ischemic colitis at the splenic flexure. Which of the following would most likely predispose this patient to ischemic colitis: Increased splanchnic blood flow following a large meal Essential hypertension Obstruction of the abdominal aorta following surgery Hyperreninemic hyperaldosteronism secondary to type II diabetes mellitus Juxtaglomerular cell tumor

C, Ischemic Colitis Ischemic colitis results from low perfusion of the bowel, most often from atherosclerosis of the superior mesenteric artery (SMA) or systemic hypotension. A partial obstruction of the abdominal aorta would reduce blood flow to the SMA and the inferior mesenteric artery (IMA) and could prompt ischemic colitis when physiologic demand increased post-prandially. Ischemic colitis is most common in elderly patients. It often occurs in watershed areas between the blood supply of two different arteries. Ischemic colitis most frequently occurs at the splenic fixture (the border area between the blood supply of the SMA and IMA) and the distal colon. Symptoms include abdominal pain, bloody diarrhea, and weight loss. Lyon et al. review abdominal pain in the elderly. Suspect intestinal or colonic ischemia in patients with acute abdominal pain and bloody stools. Low-flow states (e.g., cardiogenic shock, sepsis, dialysis, hypovolemia) predispose susceptible patients to ischemia. Elder et al. review the risk factors, pathophysiology, and clinical manifestations of ischemic colitis. Risk factors mentioned include hypotension, hypovolemia, and major vascular surgical procedures. Illustration A shows 'thumb printing' of the affected colon upon barium enema in ischemic colitis. Incorrect Answers: Answer 1: Increased splanchnic blood flow following a large meal is unlikely to lead to bowel ischemia. Answer 2: Essential hypertension is a factor in peripheral vascular disease and has been shown to be associated with ischemic colitis but is not the most likely cause compared to aortic surgery. Answer 4: Hypereninemic hyperaldosteronism involves excess secretion of both renin and aldosteronism. Increased levels of both hormones would lead to hypertension. Answer 5: A juxtaglomerular cell tumor is a rare cancer of the kidney that typically secretes renin. Increased levels of renin would lead to hypertension.

A 72-year-old man presents to the ED complaining of worsening abdominal pain over the last few hours. He also reports nausea, but denies fever, vomiting, or changes in the appearance of his bowel movements. His medical history is significant for type 2 diabetes mellitus, hypertension, coronary artery disease, stroke, atrial fibrillation, and peptic ulcer disease. Due to his recurrent bleeding peptic ulcers, he does not take warfarin. His surgical history is significant for an appendectomy as a child. His medications include metformin, lisinopril, metoprolol, and omeprazole. He has a 50-pack-year history of smoking. His temperature is 37.6 C (99.7 F), blood pressure is 146/80 mm Hg, pulse is 115/min, and respiratory rate is 20/min. On physical exam, he is in acute distress due to the pain. Pulmonary auscultation reveals scattered wheezes and decreased air entry. His heart rate is irregularly irregular, with no murmurs, rubs or gallops. Abdominal exam is significant for decreased bowel sounds and diffuse tenderness. Initial laboratory evaluation is as follows: Na 138 mEq/L, Cl 101 mEq/L, HCO3 12 mEq/L, BUN 21 mg/dL, Cr 0.9 mg/dL, glucose 190 mg/dL, amylase 240 U/L (normal < 65 U/L). What is the most likely diagnosis in this patient? Peptic ulcer perforation Acute cholecystitis Acute mesenteric ischemia Diabetic ketoacidosis Acute pancreatitis

C, Mesenteric Ischemia This patient has an anion gap metabolic acidosis - decreased bicarbonate with an increased anion gap (138-(101+12)=25, normal=6-12) - most likely the result of acute mesenteric ischemia. Patients with atrial fibrillation without anticoagulation are at risk for a variety of embolic phenomena, including acute mesenteric ischemia. Patients with bowel ischemia present with acute abdominal pain and nausea with hypoactive bowel sounds and pain out of proportion to abdominal physical exam findings. Atherosclerotic disease is another risk factor for bowel ischemia. The ischemic bowel tissue resorts to anaerobic metabolism, resulting in the production of lactic acid, which gives rise to metabolic acidosis with an anion-gap. Lyon and Clark review the diagnosis of acute abdominal pain in older adults. The physical examination can be misleadingly benign, even with serious conditions such as mesenteric ischemia. The elderly are more susceptible to acute cholecystitis and diverticulitis. In older patients with appendicitis, there are increased rates of perforation and mortality. Medication use, gallstones, and alcohol use increase the risk of pancreatitis. Bowel obstructions, usually caused by adhesions or malignancy, are more common in the elderly and often require surgery. Sise reviews the diagnosis of acute mesenteric ischemia. The 4 major types of acute mesenteric ischemia are acute superior mesenteric artery thromboembolic occlusion, mesenteric arterial thrombosis, mesenteric venous thrombosis, and nonocclusive mesenteric ischemia. Prompt diagnosis requires history and physical examination, a high index of suspicion, and early contrast CT scanning. Selective use of nonoperative therapy has an important role in nonocclusive mesenteric ischemia of the small bowel and colon. Illustration A demonstrates a contrast CT scan of a distal occlusion of the superior mesenteric artery. Illustration B depicts an angiogram of a proximal occlusion of the superior mesenteric artery. Incorrect Answers: Answer 1: Peptic ulcer perforation would cause fever and peritoneal signs on exam and would be unlikely to cause lactic acidosis early on. Answer 2: Acute cholecystitis would present with fever and right upper quadrant pain and tenderness without significant lactic acidosis. Answer 4: Diabetic ketoacidosis can present with anion-gap metabolic acidosis, abdominal pain, and nausea; but is unlikely in type 2 diabetics, and occurs with glucose levels of 250 to 600 mg/dL. Answer 5: False positive amylase levels can occur with bowel ischemia.

A 78-year-old male presents to the emergency room with crampy abdominal pain and bloating. His last bowel movement was three days ago and he has not passed gas over the same time period. His past medical history is notable for Alzheimer's disease, hypertension, and stable angina. He takes donezepil, aspirin, enalapril, and metoprolol. His temperature is 100.8°F (38.2°C), blood pressure is 90/60 mmHg, pulse is 110/min, and respirations are 24/min. On examination, his abdomen is notably distended and tender to palpation. A frontal abdominal radiograph is shown in Figure A. Which of the following is the most likely diagnosis in this patient? Intussusception Cecal volvulus Sigmoid volvulus Acute viral gastroenteritis Acute appendicitis

C, Volvulus The most likely diagnosis in this case is a volvulus of the sigmoid colon. A sigmoid volvulus has a characteristic appearance of an air-filled dilated loop of large bowel with an apex in the right upper quadrant, as seen in this patient's abdominal radiograph. A volvulus occurs when a loop of intestine twists around itself and its mesentery, leading to bowel obstruction and strangulation. Sigmoid volvulus is the most common type of volvulus and typically affects elderly patients. Cecal volvulus is less common and typically affects a younger population. Figure A demonstrates the characteristic appearance of a sigmoid volvulus on an anterior-posterior abdominal radiograph. Note the presence of an air-filled loop of large bowel in the mid abdomen with the apex in the right upper quadrant. This is sometimes referred to as "coffee bean sign" or "omega loop sign" due to its resemblance to a coffee bean or the Greek letter omega. Incorrect Answers: Answer 1: Intussusception is the most common form of bowel obstruction in children and involves the terminal ileum telescoping into the proximal large bowel. It can be visualized on ultrasound as a "donut sign" or air-contrast barium enema as a "coil-spring sign." Answer 2: Cecal volvulus is the second most common type of intestinal volvulus after sigmoid volvulus. It presents with features of proximal large bowel obstruction (crampy abdominal pain, vomiting, and abdominal distension). Answer 4: Viral gastroenteritis is a common cause of diarrhea and nausea affecting all age groups. The absence of fever and diarrhea makes an infectious etiology of this patient's symptoms unlikely. Answer 5: Acute appendicitis typically presents in younger individuals with nausea, vomiting, fever, right lower quadrant abdominal pain. Bullet Summary: Sigmoid volvulus arises when the sigmoid colon twists around its mesentery, causing strangulation and intestinal blockage.

A 4-day-old female presents with her parents to the pediatrician for a routine visit. The parents report that she was feeding well until this morning, when she had several episodes of vomiting. The parents describe the vomitus as "bright green" and report that the patient has shown little interest in feeding since then. She was born at 36 weeks gestation to a 37-year-old gravida 2. The pregnancy was complicated by gestational diabetes, and all prenatal testing was unremarkable. The patient has not yet regained her birth weight and is in the 46th percentile for height and the 36th percentile for weight. The patient's temperature is 99.2°F (37.3°C), blood pressure is 68/46 mmHg, pulse is 132/min, and respirations are 32/min. On physical exam, the patient is in mild distress. She has no dysmorphic features. Her abdomen is distended and firm without guarding. Bowel sounds are hypoactive. The patient's abdominal radiograph can be seen in Figure A, and the patient's upper gastrointestinal series can be seen in Figure B. Which of the following is the most likely etiology of this patient's condition? Duodenal atresia Hirschsprung disease Intestinal malrotation Jejunal atresia Meconium ileus

C, Volvulus This patient presents with bilious vomiting and upper gastrointestinal series demonstrating the "corkscrew" appearance of the distal duodenum and proximal jejunum, which is consistent with a diagnosis of intestinal malrotation. Intestinal malrotation most commonly presents in newborns with bilious emesis and increases the risk of a midgut volvulus. Abdominal radiographs are rarely diagnostic but are useful in excluding bowel perforation. Upper gastrointestinal series is the diagnostic test of choice and demonstrates an abnormally placed duodenum with a ligament of Treitz on the right side of the abdomen and a "corkscrew" appearance of the distal duodenum and proximal jejunum. Figure A demonstrates the nonspecific findings of intestinal malrotation on abdominal radiograph, including a paucity of air in the abdomen. Figure B demonstrates the classic findings of intestinal malrotation on upper gastrointestinal series, including a right-sided ligament of Treitz and a "corkscrew" appearance of the distal duodenum. Incorrect Answers: Answer 1: Although duodenal atresia presents with bilious vomiting, the "double bubble" sign would be seen on abdominal radiograph, which reflects air trapping in both the stomach and first portion of the duodenum. Answer 2: Hirschsprung disease may also cause bilious vomiting, but abdominal radiograph would demonstrate proximal colonic distension with little air in the rectosigmoid colon. Hirschsprung disease is also well visualized on contrast enema, on which a transition point can be seen between the proximal dilated colon and the narrow sigmoid. Answer 4: Jejunal atresia can also lead to bilious vomiting, although a "triple bubble" sign would be seen on abdominal radiograph with air additionally filling the proximal jejunum. Answer 5: Meconium ileus may present with bilious emesis, but multiple dilated loops of bowel would be seen on abdominal radiograph. Meconium ileus is better characterized on contrast enema, on which a microcolon can be visualized. Bullet Summary: Intestinal malrotation presents in newborns with bilious emesis, an upper gastrointestinal series demonstrating a misplaced duodenum, and a "corkscrew" appearance of the distal duodenum and proximal jejunum.

A 65-year-old obese female presents to the emergency room complaining of severe abdominal pain. She reports pain localized to the epigastrium that radiates to the right scapula. The pain occurred suddenly after a fast food meal with her grandchildren. Her temperature is 100.9°F (38.2°C), blood pressure is 140/85 mmHg, pulse is 108/min, and respirations are 20/min. On examination, she demonstrates tenderness to palpation in the epigastrium. She experiences inspiratory arrest during deep palpation of the right upper quadrant but this exam finding is not present on the left upper quadrant. A blockage at which of the following locations is most likely causing this patient's symptoms? Common hepatic duct Ampulla of Vater Cystic duct Pancreatic duct of Wirsung Common bile duct

C,Acute Cholecystitis The most likely diagnosis in this patient is acute cholecystitis. This condition is caused by prolonged blockage of the cystic duct leading to distention and inflammation within the gallbladder. Acute cholecystitis is caused by a blockage of the cystic duct by a gallstone, leading to distention and inflammation of the gallbladder. This condition commonly affects obese females aged 40 to 50 years old and frequently occurs after a fatty meal. Although cholecystic pain classically occurs in the right upper quadrant, 1/3 of cases In elderly individuals (such as the patient in this vignette) will present with epigastric pain. The presence of Murphy's sign (inspiratory arrest during deep palpation of the right upper quadrant) is specific for acute cholecystitis. Ultrasound findings will often demonstrate pericholecystic fluid, a thickened gallbladder wall, and the presence of gallstones. If the ultrasound is equivocal, a HIDA scan is used to confirm the diagnosis and will show an absent gallbladder. Illustration A depicts the anatomy of the biliary tree. Incorrect Answers: Answer 1: The common hepatic duct is formed by the junction of the right and left hepatic ducts, which drain bile from both sides of the liver. The common hepatic duct then joins the cystic duct that arises from the gallbladder. A blockage of the common hepatic duct would therefore not block drainage from the gallbladder and would not cause acute cholecystitis. Answer 2: The common bile duct merges with the pancreatic duct of Wirsung to form the ampulla of Vater which drains into the duodenum. A blockage at the ampulla of Vater will block secretions from both the common bile duct and pancreatic duct of Wirsung. Answer 4: The pancreatic duct joins the pancreas with the common bile duct to drain secretions from the exocrine pancreas. A blockage here may cause pancreatic inflammation but would not affect bile drainage from the liver and gallbladder via the common bile duct. Answer 5: The common bile duct is formed at the junction of the common hepatic duct and cystic duct. A blockage at the common bile duct would cause backup of secretions into the gallbladder and liver. This condition is known as choledocholithiasis and is differentiated from acute cholecystitis by the presence of liver damage. Bullet Summary: Acute cholecystitis is caused by blockage of the cystic duct leading to gallbladder distention and inflammation; it classically presents with right upper quadrant pain, fever, leukocytosis, Murphy's sign, and pericholecystic fluid and a thickened gallbladder wall on ultrasound.

An 11-year-old girl presents to the emergency department with a 12-hour history of severe abdominal pain. She says that the pain started near the middle of her abdomen and moved to the right lower quadrant after about 10 hours. Several hours after the pain started she also started experiencing nausea and loss of appetite. On presentation, her temperature is 102.5°F (39.2°C), blood pressure is 115/74 mmHg, pulse is 102/min, and respirations are 21/min. Physical exam reveals rebound tenderness in the right lower quadrant. Raising the patient's right leg with the knee flexed significantly increases the pain. Which of the following is the most common cause of this patient's symptoms in children? Fecalith obstruction Ingestion of indigestible object Lymphoid hyperplasia Meckel diverticulum Parasitic infection

C,Appendicitis This child with nausea, fever, and abdominal pain that progressed from the periumbilical region to the right lower quadrant most likely has appendicitis. The most common cause of appendicitis in children is lymphoid hyperplasia. Appendicitis is most commonly seen in patients between the ages of 10-30. The most common etiology for this disease in children is lymphoid hyperplasia. This can occur post-immunization or after infection with adenovirus, measles, or other viruses with fecal-oral transmission patterns. In adults, the most common cause is fecalith obstruction because this object can create a dead-end loop in the appendix that allows for slow bacterial proliferation. Other causes include ingestion of hard objects such as seeds or infections with parasites such as pinworm. Incorrect Answers: Answer 1: Fecalith obstruction is the most common cause of appendicitis in adults but not in children. Answer 2: Ingestion of indigestible object can occlude the appendix; however, this is not the most common cause of appendicitis. Answer 4: Meckel diverticulum is a congenital structural abnormality that can also present with right lower quadrant pain; however, it is not a common cause of appendicitis. Answer 5: Parasitic infection with pinworm can lead to appendicitis; however, it is not commonly seen in the United States. Bullet Summary: Lymphoid hyperplasia after viral infection or immunization is the most common cause of appendicitis in children.

A 17-year-old woman presents to the emergency department with abdominal and pelvic pain. She states it started 3 days ago and it has been getting gradually worse. She states it is diffuse and is located over her abdomen, pelvis, and inside her vagina. She also endorses vaginal pruritus and a discharge from her vagina. The patient works in an ice cream parlor and is sexually active with multiple different partners. Her temperature is 98.0°F (36.7°C), blood pressure is 122/80 mmHg, pulse is 82/min, respirations are 15/min, and oxygen saturation is 98% on room air. Physical exam is notable for a foul smelling vagina with a thin, white discharge. Her abdomen is diffusely tender. The patient is noted to be itching her vagina during the exam. Which of the following is the most appropriate initial step in management? Cervical swab and culture CT abdomen/pelvis KOH prep Urine hCG Wet mount

D, Acute Abdomen Female patients of reproductive age who present with uropelvic or abdominal complaints should always be screened for pregnancy with a urine hCG before further more invasive tests. The differential diagnosis of abdominal pain is very broad in a female patient. Possible life-threatening diagnosis can include appendicitis, mesenteric ischemia, cholecystitis, cholangitis, perforated viscus, ovarian torsion, volvulus, small bowel obstruction, ectopic pregnancy, and heterotopic pregnancy. In young female patients, the workup should always start with a urine hCG to rule out an obvious pregnancy. Even if pregnancy is not considered to be in the differential, if a patient is pregnant it may affect subsequent care and imaging studies (such as ordering CT scans considering the radiation exposure). After a urine hCG results, then further more invasive testing can be performed. Incorrect Answers: Answer 1: Cervical swab and culture may be indicated in the workup of this patient if no other etiology of her symptoms is discovered. Her thin, white vaginal discharge is more suggestive of a physiologic discharge. Answer 2: CT abdomen/pelvis would be indicated after initial laboratory tests are negative and the patient has focal tenderness. It would be appropriate in this patient; however, her urine hCG should result first. Answer 3: KOH prep is generally not indicated in the workup of candida vaginitis which is a clinical diagnosis. It presents with a thick, white discharge with intense pruritus. This diagnosis is possible in this patient; however, she should have a urine hCG performed first. Answer 5: Wet mount would be indicated to diagnose trichomoniasis which presents with a strawberry cervix, pruritus, and a frothy and green discharge. The treatment is metronidazole. Bullet Summary: A urine hCG should be ordered in women of reproductive age with abdominal or pelvic symptoms.

A 70-year-old male presents to the emergency department with fever and productive cough that has progressed rapidly over the past day. On exam, he is very ill appearing and his vitals demonstrate T: 39 deg C, HR: 95 bpm, BP: 80/40 mmHg, RR: 15, SaO2: 93%. A CBC is obtained which demonstrates a white blood cell count (WBC) of 14,000. A chest radiograph is obtained which is demonstrated in Figure A. The patient is subsequently diagnosed with severe sepsis secondary to pneumonia and is transferred to the ICU after intubation, appropriate antibiotic therapy, and resuscitation measures are initiated. The patient improves steadily over the subsequent days in the ICU with improving vitals and decreasing WBC. However, on hospital day 4 he develops a fever to 41 deg C, and his WBC elevates to 16,000. On exam, he appears to withdraw in pain when his abdomen is palpated. Liver function tests and amylase/lipase are ordered and shown to be within normal limits. An abdominal ultrasound is obtained which is demonstrated in Figure B. What is the most likely cause of this patient's current presentation? Cholelithiasis Cholangitis Acute calculous cholecystitis Acute acalculous cholecystitis Acute pancreatitis

D, Acute Cholecystitis This patient is presenting with acute acalculous cholecystitis. Acute acalculous cholecystitis is the acute inflammation of the gallbladder in the absence of gallstones, often observed in very ill patients with sepsis, trauma, or severe burns. Acute acalculous cholecystitis can complicate severe illness or major trauma and surgery. It is defined as inflammation of the gallbladder without evidence of gallstones. It is thought to be caused by bile stasis which occurs in severe illness due to lack of oral intake. Acute acalculous cholecystitis is very dangerous and carries a mortality rate of approximately 30%, most likely due its ability to progress rapidly to perforation and gangrene. Diagnosis is challenging as most patients are already very ill and often cannot communicate effectively. However, commonly observed findings are fever, elevated WBC, and right sided abdominal pain. Diagnosis is usually made by ultrasound which shows gallbladder inflammation without evidence of gallstones. Treatment is immediate cholecystectomy if patients can tolerate surgery or percutaneous drainage via cholecystostomy. Treinen et al. review the risk factors and surgical treatment options for patients with acute acalculous cholecystitis. They state that acute acalculous cholecystitis is characterized by severe gallbladder inflammation without cystic duct obstruction. They find that patients that are healthy enough to tolerate laparoscopic cholecystectomy should undergo laparoscopic cholecystectomy early in the course of the disease. In critically ill patients, patients with multiple comorbidities, or who are poor surgical candidates, percutaneous cholecystostomy may be the safest and most successful intervention. Kirkegård et al. examine the effectiveness of percutaneous cholecystostomy as definitive treatment for acute acalculous cholecystitis. They perform an observation study and find percutaneous cholecystostomy to be successful as a definitive treatment option in the majority of patients with acute acalculous cholecystitis. It was associated with a low rate of mortality and subsequent cholecystectomy. Figure A is a chest radiograph demonstrating a lobar pneumonia, the cause of severe sepsis in this patient. Figure B demonstrates the ultrasound findings observed in acute acalculous cholecystitis. Note the gallbladder wall thickening without evidence of gallstone. Illustration A is a diagram demonstrating the pathogenesis of acute acalculous cholecystitis. Illustration B is a gross specimen of a gangrenous gallbladder resulting from acute acalculous cholecystitis. Incorrect answers: Answer 1: Cholelithiasis (gallstones) typically present as biliary colic, sharp RUQ pain following large meals. This is not consistent with this patient's severe presentation. Answer 2: Cholangitis is characterized by Charcot's triad of jaundice, fever, and RUQ pain. While this patient has fever and RUQ pain, he is not jaundiced as would be expected in cholangitis. Answer 3: Acute calculous cholecystitis would be expected to demonstrate gallstones on ultrasound. Answer 5: This patient's amylase and lipase are normal making acute pancreatitis unlikely.

A 38-year-old female is brought in to the emergency room by her husband for fever and right upper quadrant pain. She states that 2 days ago she noticed a 7/10 sharp, stabbing pain that was intermittent. She took ibuprofen at that time, which provided some relief. However, the pain got progressively worse the next day and she developed a fever of 100.8°F (38.2°C). Her temperature is 101.3°F (38.5°C), blood pressure is 150/84mmHg, pulse is 104/min, and respirations are 15/min. A physical examination demonstrates right upper quadrant tenderness with some diffuse voluntary guarding. An eye examination is demonstrated in Figure A. Laboratory findings are shown below. Hemoglobin: 14 g/dL Hematocrit: 38 % Leukocyte count: 21,000/mm^3 with neutrophilic predominance Platelet count: 180,000/mm^3 Serum: Na+: 137 mEq/L Cl-: 96 mEq/L K+: 4.1 mEq/L HCO3-: 25 mEq/L BUN: 17 mg/dL Glucose: 130 mg/dL Creatinine: 0.8 mg/dL Thyroid-stimulating hormone: 2.1 µU/mL Ca2+: 9.1mg/dL AST: 430 U/L ALT: 387 U/L Lipase: 59 U/L What is the most likely explanation for this patient's presentation? Acute inflammation of the gallbladder Autodigestion of the pancreas Autoimmune destruction of the intralobular bile ducts Infection of the biliary tree Viral infection of the liver

D, Acute Cholecystitis This patient likely has ascending cholangitis, which is an infection of the biliary tree, as demonstrated by the presence of the Charcot triad: jaundice, fever, and right upper quadrant pain. Ascending cholangitis is inflammation and infection of the bile duct that is usually due to an obstruction that leads to stasis and subsequent ascension of bacteria from the duodenum. Patients may complain of abdominal pain (often in the right upper quadrant), fever, and jaundice, which makes up the Charcot triad. Symptoms such as hypotension (or shock) and altered mental status in addition to the triad make up the Reynold pentad, which indicates disease progression and the development of sepsis. Laboratory studies will often show features of acute inflammation (e.g., elevated white cell count) and abnormal liver function tests that are often consistent with obstruction (e.g., high bilirubin and alkaline phosphatase), though in early stages it may resemble those seen in hepatitis (e.g., high AST and ALT). Diagnosis often involves imaging studies such as ultrasound, which is usually the best first test, and endoscopic retrograde cholangiopancreatography (ERCP), which is the gold standard for biliary obstruction. Treatment often involves fluids, antibiotics, and endoscopic relief of the obstruction. Figure A is a clinical image demonstrating scleral icterus (yellowing of the conjunctiva), a sign of jaundice. Incorrect Answers: Answer 1: Acute inflammation of the gallbladder describes cholecystitis, which often occurs as a result of a gallstone at the neck of the bladder. Although patients may complain of right upper quadrant pain and fever, patients rarely present with jaundice seen in ascending cholangitis. Answer 2: Autodigestion of the pancreas describes pancreatitis, which often presents with significant epigastric pain that radiates to the back. Gallstones are one of the common etiologies of acute pancreatitis in the United States. However, this patient's lipase is within normal limits. Answer 3: Autoimmune destruction of the intralobular bile ducts describes primary biliary cholangitis, which is a biliary tract disease often seen in middle-aged women with other autoimmune diseases. Patients may present with pruritis, jaundice, hepatosplenomegaly, or dark urine. Patients usually do not complain of fever or acute-onset right upper quadrant pain. Answer 5: Viral infection of the liver suggests viral hepatitis, which can present with fever/flu-like symptoms, jaundice, and right upper quadrant pain. However, patients with viral hepatitis often have AST and ALTs in the thousands instead of in the hundreds seen in this patient, which is more consistent with an early obstructive picture. Bullet Summary: Ascending cholangitis often presents with the Charcot triad, which includes fever, right upper quadrant pain, and jaundice.

A 37-year-old woman with a past medical history diabetes, high cholesterol, hypertension and anxiety complains of rapid-onset, mid-epigastric pain with radiation to the back. She states that she has had a few of these episodes in the past, but this is the worst so far. Her pain is somewhat relieved by sitting forward. Physical examination notes epigastric tenderness without guarding or rebound. Figure A is also noticed on physical examination. Laboratory studies show elevated amylase and lipase. She denies use of alcohol, tobacco, and illicit drugs. A RUQ ultrasound was performed and found to be negative and total and direct bilirubin are normal. Urine toxicology returns negative results and her blood alcohol level is determined to be 0. What is the most likely cause of her current symptoms? Alcoholism Scorpion sting Biliary tract obstruction Hypertriglyceridemia Hypercalcemia

D, Acute Pancreatitis This patient is experiencing an episode of acute pancreatitis. Hypertriglyceridemia should be very high on the differential when a patient denies alcohol use, there is no evidence of cholelithiasis, and has xanthomas on physical examination. Acute pancreatitis is a life threatening disease that is caused by leaking pancreatic enzymes that autodigest the pancreas. Over 70% of cases are caused by alcoholism and gallstones. If patients do not have gallstones or a history of alcohol use, other causes must be explored. Acute pancreatitis can be caused by medications, hypertriglyceridemia, hypercalcemia, recent ERCP, trauma, infection, and even scorpion stings. If patients have xanthomas or xanthelasmas, it is likely that they have hypertriglyceridemia, which may be the cause of their acute pancreatitis. Patients often present with severe epigastric pain and elevated amylase and lipase. Acute treatment includes IV fluids, bowel rest, and nasogastric decompression. Figure A demonstrates eruptive xanthomas which, when seen, should be associated with high triglyceride levels. Illustration A is an overview image of how gallstones can block the common bile duct and cause inflammation of the pancreas. Illustration B depicts an axial CT in a patient with acute exudative pancreatitis showing extensive fluid collections surrounding the pancreas. Incorrect Answers: Answers 1-3, 5: Although each of these are able to cause acute pancreatitis, hypertriglyceridemia is the most likely cause in the above vignette.

A 33-year-old man presents to the emergency department after he is found passed out next to his car. The patient's car was found rammed into a telephone pole. The patient was found napping outside of his car on a bench nearby. Upon arrival, the patient is too intoxicated to offer a useful history. His past medical history is notable for alcohol and IV drug abuse. His temperature is 97.6°F (36.4°C), blood pressure is 124/94 mmHg, pulse is 105/min, respirations are 12/min, and oxygen saturation is 98% on room air. The patient has a Glasgow coma scale of 12 and is not intubated. He is observed in the emergency department with a plan for discharge when clinically sober. The patient is evaluated 7 hours later, and his heart rate is 130/min and blood pressure is 95/60 mmHg. The finding in Figure A is present. He appears obtunded and only arouses to sternal rub. Which of the following is the most appropriate next step in management? CBC CT abdomen/pelvis Exploratory laparotomy Focused assessment with sonography in trauma (FAST) exam Observation and serial abdominal exams

D, Acute Pancreatitis This patient is presenting after a motor vehicle collision with instability of his vitals and Cullen sign on physical exam, suggesting a diagnosis of a retroperitoneal hemorrhage or bleeding from blunt abdominal trauma. In the setting of this patient's unstable vitals, a Focused assessment with sonography in trauma (FAST) exam to assess​ for bleeding is indicated. Blunt abdominal trauma can damage internal organs that can lead to intraabdominal bleeding. For most patients, a primary survey, secondary survey, and initial FAST exam can pick up bleeding. However, in some cases, there can be bleeding that occurs later on. For this reason, patients should be observed when there is a serious mechanism of abdominal trauma. Two physical exam findings that can suggest retroperitoneal and intraabdominal bleeding include Grey-Turner and Cullen sign. Grey-Turner sign refers to bruising of the flanks (and is indicative of a retroperitoneal bleed), and Cullen sign refers to bruising around the umbilical region. Cullen sign is associated with acute pancreatitis, intraabdominal bleeding, and retroperitoneal bleeding. Figure A demonstrates Cullen sign with bruising around the umbilical region. Incorrect Answers: Answer 1: CBC should be drawn at this time to trend this patient's hemoglobin and to see if he is bleeding. It would not be the most appropriate initial step in management because drawing this lab and waiting for results takes time in an unstable patient. Additionally, the hemoglobin concentration can be initially normal in the setting of an acute bleed despite the significant blood loss. Answer 2: CT abdomen/pelvis would be appropriate if this patient was stable; however, his unstable vitals make CT scan an inappropriate choice. Answer 3: Exploratory laparotomy may be indicated after a FAST exam and fluid or blood product resuscitation is started. If the patient is not responsive or the FAST exam is positive this would be appropriate. Answer 5: Observation and serial abdominal exams are inappropriate as this patient is demonstrating Cullen sign and has unstable vitals. Bullet Summary: Cullen sign suggests late retroperitoneal or intraabdominal bleeding.

A 39-year-old woman is brought to the emergency room by her fiancé for severe abdominal pain for the past 5 hours. She was watching TV after dinner when she felt a sudden, sharp, 10/10 pain at the epigastric region that did not go away. Ibuprofen also did not help. She reports recurrent abdominal pain that would self-resolve in the past but states that "this one is way worse." Her past medical history is significant for diabetes and an appendectomy 2 years ago. The patient endorses nausea and 1 episode of emesis, but denies fevers, chills, chest pain, shortness of breath, diarrhea, constipation, urinary symptoms, paresthesia, or weakness. She used to smoke marijuana in college and drinks about 2 beers a week. A physical examination demonstrates an overweight woman in acute distress with diffuse abdominal tenderness. Her vitals are within normal limits. Laboratory values are shown below: Hemoglobin: 12 g/dL Hematocrit: 34% Leukocyte count: 4,900/mm^3 with normal differential Platelet count: 160,000/mm^3 Serum: Na+: 138 mEq/L Cl-: 98 mEq/L K+: 4.8 mEq/L HCO3-: 25 mEq/L Glucose: 123 mg/dL Ca2+: 6.9 mg/dL AST: 387 U/L ALT: 297 U/L ALP: 168 U/L Lipase: 650 U/L (Normal 0 - 160 U/L) What is the best next step in the workup of this patient? Abdominal computed tomography with IV contrast Abdominal computed tomography without IV contrast Abdominal magnetic resonance imaging Abdominal ultrasound Upright chest and abdominal radiograph

D, Acute Pancreatitis This patient is presenting with acute pancreatitis (e.g., sudden-onset, severe abdominal pain with elevated lipase) secondary to probable cholelithiasis (e.g., female, body habitus and recurrent abdominal pain). An abdominal ultrasound would be the most appropriate next step in diagnosing the condition. Acute pancreatitis is secondary to autodigestion of the pancreas by pancreatic enzymes. The most common etiology of acute pancreatitis in the United States includes alcoholic pancreatitis and gallstone pancreatitis. Diagnosis of acute pancreatitis can be done clinically as only 2 out of the 3 criteria need to be met: acute epigastric pain often radiating to the back, increased serum amylase or lipase to 3x upper limit of normal, or characteristic imaging findings. Ultrasound may demonstrate presence of gallstones at the gallbladder, peripancreatic fluid and pancreatic calcifications. Management includes resuscitation if the patient is unstable, intravenous fluids, pain management, and bowel rest. Incorrect Answers: Answer 1: Abdominal computed tomography (CT) with IV contrast used to be the standard of care in terms of diagnosis but is no longer recommended at initial presentation unless there is diagnostic uncertainty. This patient is already presenting with 2 out of the 3 diagnostic criteria and thus does not need additional abdominal CT. Answer 2: Abdominal computed tomography without IV contrast is the standard of diagnosis for renal stones, which usually present with acute onset colicky pelvic pain that may radiate to the groin. This patient's pain is not characteristic of renal stones. Answer 3: Abdominal magnetic resonance imaging has a higher sensitivity for the diagnosis of early acute pancreatitis compared to contrast-enhanced abdominal CT scan. However, due to its long scanning time and dependency on the operator, it is not the first choice in the workup. Answer 5: Upright chest and abdominal radiograph would be appropriate if one is concerned about a perforated viscus. However, this patient's stable vital signs and presentation are more suggestive of gallstone pancreatitis. Bullet Summary: Acute pancreatitis can be diagnosed clinically via specific diagnostic criteria; abdominal ultrasound may be useful in determining the etiology if one suspects gallstone pancreatitis.

A 39-year-old woman is admitted to the hospital with 1 day of abdominal pain and vomiting. Her medical history includes peptic ulcer disease, chronic constipation, and bipolar disorder. She denies any alcohol, tobacco, or illicit drug use. Physical exam is notable for epigastric tenderness. Her temperature is 101.5°F (38.6°C), blood pressure is 100/65 mmHg, pulse is 105/min, and respirations are 20/min. Abdominal CT scan is performed (Figure A). Which of the following medications was this patient most likely taking prior to admission? Amoxicillin Lithium Pantoprazole Valproic acid Simvastatin

D, Acute Pancreatitis This woman with a history of peptic ulcer disease and bipolar disorder, who presents with a CT scan and symptoms suggesting pancreatitis, was most likely taking valproic acid, which is both a common treatment for bipolar disorder and a cause of acute pancreatitis. Acute pancreatitis is inflammation of the pancreas, often involving release of pancreatic enzymes and autodigestion of the pancreas. The most common causes are heavy alcohol use (alcoholic pancreatitis) and gallstone blockage of the pancreatic duct (gallstone pancreatitis). It can also be caused by several medications, including valproic acid, antibiotics (especially sulfa antibiotics), antivirals (especially protease inhibitors and nucleoside reverse transcriptase inhibitors, both which are treatments for HIV), statins, and antihypertensives (especially thiazides and ACE inhibitors), among others. Classic symptoms include acute onset epigastric abdominal pain and tenderness, nausea, vomiting, and fever. Labs show elevated pancreatic enzymes (amylase and lipase) in the serum. The diagnosis is made based on the combination of classic symptoms plus either elevated lipase (> 3x the normal limit) or a CT scan showing pancreatitis. Treatment involves suspending the offending agent and supportive care (NPO, IV fluids, and pain medications). Figure/Illustration A is an abdominal CT scan demonstrating an enlarged pancreas (red arrow) with indiscriminate margins (due to inflammation), which suggests pancreatitis. Incorrect Answers: Answer 1: Amoxicillin is not known to cause pancreatitis. It is part of the triple therapy treatment regimen for peptic ulcer disease caused by Helicobacter pylori, a disease which can be asymptomatic or present with recurrent post-prandial epigastric pain and nausea. Answer 2: Ibuprofen can exacerbate or cause peptic ulcers, which also present with epigastric pain and tenderness but would not explain this patient's fever or CT scan findings. Ibuprofen is not associated with pancreatitis, the more likely diagnosis here. Answer 3: Lithium (a treatment for bipolar disorder) can cause renal failure, hypothyroidism, and nephrogenic diabetes insipidus, and can accumulate to toxic levels (as it has a narrow therapeutic window). However, it is not known to cause pancreatitis. Answer 5: Simvastatin (and statins in general) can cause acute pancreatitis. However, this patient's medical history does not suggest any indication for her to be on a statin (e.g., hyperlipidemia or a prior myocardial infarction). Bullet Summary: Acute pancreatitis can be caused by heavy alcohol use, gallstones, or numerous medications (including valproic acid), and classically presents with epigastric pain, vomiting, fever, and enlarged and inflamed pancreas on CT.

A 24-year-old woman presents to the emergency department with sudden onset of severe abdominal pain. She states that the pain first started around her belly button and then migrated to the right lower abdomen. She complains of nausea and non-bloody, non-bilious vomiting. The patient's temperature is 101.3°F (38.5°C), blood pressure is 109/70 mmHg, pulse is 88/min, and respirations are 15/min. On physical examination, there is tenderness at the right lower quadrant with guarding. Which of the following is the next best step? Abdominal radiograph Computed tomography Magnetic resonance imaging Pregnancy test Surgical exploration

D, Appendicitis The patient is presenting with classic features of appendicitis such as right lower abdominal pain, nausea, and vomiting. Women of childbearing age should receive a pregnancy test prior to more invasive tests. Clinical signs and symptoms of appendicitis include migratory right lower quadrant pain, fever, nausea, vomiting, anorexia, rebound tenderness, and guarding. Computed tomography (CT) is the preferred modality to increase the diagnostic specificity for appendicitis. All women presenting with abdominal pain, even with very high suspicion for appendicitis, should receive a pregnancy test prior to exposure to radiation (e.g., radiographs or CT scans) or invasive tests (e.g., surgical exploration). If the pregnancy test is positive, ultrasound or magnetic resonance imaging can be used instead of radiation-based imaging. Incorrect Answers: Answer 1: Abdominal radiograph is not recommended in the work-up of suspected appendicitis as it poorly visualizes the appendix. Additionally, a radiograph performed on a patient prior to obtaining a negative pregnancy test could potentially expose a fetus to radiation that may cause irreversible damage. Answer 2: Computed tomography (CT) is the preferred test for imaging in an adult suspected of appendicitis. However, this imaging modality would expose a fetus to radiation, and a negative pregnancy test should be obtained first. Answer 3: Magnetic resonance imaging (MRI) can be used in pregnant women. The rate of nondiagnostic exams with MRIs is higher than with CT, and MRIs are often less tolerated. A pregnancy test should be obtained first. If negative, CT would be the preferred imaging modality. Answer 5: Surgical exploration is warranted if imaging studies are nondiagnostic or unavailable, and suspicion is high. Bullet Summary: Women of childbearing age who present with abdominal pain should receive a pregnancy test prior to invasive tests and imaging that could expose a potential fetus to radiation.

A 23-year-old woman presents to the emergency department for vomiting and abdominal pain. The patient states that she has been unable to eat or drink for the past 24 hours without vomiting. She also complains of worsening abdominal pain that started 3 days ago. The patient has a past medical history of IV drug abuse and alcohol abuse. She is not on any current medications. Her temperature is 99.5°F (37.5°C), blood pressure is 127/68 mmHg, pulse is 120/min, respirations are 17/min, and oxygen saturation is 98% on room air. On physical exam, tenderness is elicited when the right lower quadrant of her abdomen is palpated. Deep palpation and release of the left lower quadrant of her abdomen also causes severe pain. Rectal exam reveals normal tone and stool is Guiac negative. Laboratory studies are ordered as seen below. Hemoglobin: 12 g/dL Hematocrit: 36% Leukocyte count: 11,500/mm^3 with normal differential Platelet count: 197,000/mm^3 Serum: Na+: 139 mEq/L Cl-: 100 mEq/L K+: 4.3 mEq/L HCO3-: 27 mEq/L BUN: 20 mg/dL Glucose: 67 mg/dL Creatinine: 1.1 mg/dL Ca2+: 10.2 mg/dL AST: 12 U/L ALT: 15 U/L Urine: Cocaine: positive Amphetamines: positive ß-hCG: positive Marijuana: positive Heroin: negative PCP: negative MDMA: positive Glucose: negative Ketones: negative Which of the following is the next best step in management? Chest radiograph Abdominal CT Abdominal MRI Ultrasound Colonoscopy

D, Appendicitis This patient is presenting with a positive pregnancy test and symptoms suggestive of appendicitis. The best next step in management in a pregnant patient is an ultrasound of the abdomen. Appendicitis presents with nausea, vomiting, and periumbilical abdominal pain that migrates to the right lower quadrant as the disease progresses. When pressure is applied to the left lower quadrant, symptoms can be experienced in the right lower quadrant as well. The best diagnostic test of appendicitis is a CT scan of the abdomen. In pediatric patients and pregnant patients, ultrasound is the preferred diagnostic modality as it decreases radiation exposure. Incorrect Answers: Answer 1: A chest radiograph exposes this pregnant patient to radiation and would not aid in the diagnosis. Answer 2: An abdominal CT is the best diagnostic test for appendicitis; however, it is not the preferred test in pregnant and pediatric patients due to the radiation exposure. Answer 3: An abdominal MRI could diagnose appendicitis in pediatric/pregnant patients; however, it is a slow and costly exam that is not preferred over ultrasound. Answer 5: Colonoscopy is not indicated in the diagnosis of appendicitis but is a possible diagnostic modality in colon cancer and inflammatory bowel disease. Bullet Summary: The best diagnostic test for appendicitis is an abdominal CT in regular patients and an abdominal ultrasound in pediatric/pregnant patients.

A 12-year-old boy is brought in by his mother to the emergency department. He has had abdominal pain, fever, nausea, vomiting, and loss of appetite since yesterday. At first, the mother believed it was just a "stomach flu," but she is growing concerned about his progressive decline. Vitals include: T 102.3 F, HR 110 bpm, BP 120/89 mmHg, RR 16, O2 Sat 100%. Abdominal exam is notable for pain over the right lower quadrant. What is the next best step in management in addition to IV hydration and analgesia? Abdominal CT scan with IV and PO contrast Abdominal CT scan with IV contrast Upright and supine abdominal radiographs Right lower quadrant ultrasound Abdominal MRI with gadolinium contrast

D, Appendicitis This patient presents with signs and symptoms concerning for appendicitis. Diagnosis should be approached first with a right lower quadrant ultrasound (US), with every attempt made to avoid radiation exposure to pediatric patients (Illustration A). Acute appendicitis occurs as a result of obstruction of the appendiceal orifice. This blockage leads to fluid buildup and increased intraluminal pressure. The increased pressure eventually causes decreased venous outflow, leading to edema and possible ischemia. Over time this process can lead to appendiceal rupture and peritonitis. Diagnosis in the pediatric population should first be attempted through a right lower quadrant ultrasound. If this does not yield a diagnosis, additional imaging with abdominal CT scan is appropriate. Definitive management is surgical, with laparoscopic appendectomy being the procedure of choice. Leung and Sigalet discuss the acute abdomen in children. They comment that appendicitis is the most common cause of acute abdomen in the 12-18 year age group. Analgesics were conventionally discouraged, as these interfered with the ability of serial abdominal exams to guide therapy; however, these authors discuss how this practice has now given way to pain control. Howell et al. discuss the findings from the clinical policy from the American College of Emergency Physicians. They buttress the approach to the pediatric patient of attempting a right lower quadrant abdominal ultrasound before CT scan. In particular, they highlight the role of ultrasound in confirming appendicitis but not in ruling it out. This is a reflection of a high positive predictive value, but relatively poor negative predictive value of ultrasonographic findings in acute appendicitis. Illustration A shows US showing markedly dilated appendix with thickened wall, consistent with acute appendicitis. Illustration B shows an abdominal CT scan showing a markedly dilated appendix and fat stranding consistent with acute appendicitis. Incorrect Answers Answers 1, 2: Abdominal ultrasound should be attempted to confirm acute appendicitis before exposing pediatric patients to ionizing radiation. Answer 3: Abdominal plain films would be a poor diagnostic modality for suspected acute appendicitis. Answer 5: Abdominal MRI is not routinely employed for the diagnosis of acute appendicitis in pediatric patients. It is used in pregnant patients to spare ionizing radiation to the fetus.

A 23-year-old college student presents to the emergency room with severe abdominal pain, nausea, vomiting, and anorexia for the last 24 hours. She reports that she was in her usual state of health, until yesterday when she began to have abdominal pain which she describes near her belly button. This morning, she woke up with her pain much increased, and shifted to the right lower quadrant. Vitals are T 102.0 F HR 98 bpm BP 138/94 mmHg RR 18 Sat 100%. Exam is notable for exquisite right lower quadrant pain that is worse with rebound. When the examiner presses on the left lower quadrant, the patient complains of pain in the right lower quadrant. Psoas signs is present, with a negative obturator sign. Lab studies show a white blood cell count of 15.3, beta HCG is negative. Abdominal CT with contrast is shown in Figure A. Which of the follow pathophysiologies most likely lead to this patient's condition? Telescoping of hollow viscus into adjacent segment due to a lead point resulting in vascular compromise Rotation of adnexa about ligamentous structures leading to venous and arterial compromise Implantation of zygote within the fallopian tubes leading to tubular dilation and rupture Luminal obstruction of hollow viscus by fecolith or lymphoid tissue leading to venous outflow obstruction Viral infection with hyperplasia of lymphoid tissue, and inflammation of mesenteric nodes

D, Appendicitis This patient presents with signs and symptoms of acute appendicitis. The pathophysiology of acute appendicitis involved obstruction of the orifice of the appendix, leading to increased intraluminal pressure from continued secretions, venous outflow obstruction with resultant edema, and eventually ischemia and secondary infection. Acute appendicitis is one of the most common causes of acute abdomen, and one of the most frequent indications for an emergent abdominal surgery in the world. The characteristic pain sequelae of acute appendicitis sheds light to the pathophysiology. As intraluminal pressure mounts within the vermiform appendix, it becomes engorged, and stimulates the visceral afferent nerve fibers entering the spinal cord at the T8-T10 level, leading to vague periumbilical referred pain. Well localized pain occurs later in the course, once the inflammation begins to involve the adjacent parietal peritoneum. Similarly, the presence of the the psoas sign is indicative of a retrocecal appendix, which when the psoas muscle is flexed, results in peritonitis at this point. The obturator sign is indicative of an abdominal appendix, with a similar mechanism as the psoas sign, but this time involving the inflamed, draped appendix over the right obturator internus. Howell et al. discuss the evaluation and management of patients with suspected appendicitis. They conclude that any female of childbearing age should be first screened of the possibility of pregnancy, with qualitative B-HCG being an appropriate modality. In children, acute appendicitis should be first explored with a right lower quadrant ultrasound (US). US should be used to confirm acute appendicitis, but not exclude it. In children without a definitive diagnosis after US, computed tomography is appropriate. Interestingly, Anderson et al. found that the addition of oral contrast to IV contrast in computed tomography did not significantly improve the sensitivity or specificity of the scan with regards to acute appendicitis. Korndorffer et al. discuss the recommendations regarding surgical approaches to appendectomies. In general, they conclude that laparoscopic appendectomy is a reasonable approach for uncomplicated and complicated appendicitis. Laparoscopic appendectomies has a lower rate of of postoperative wound infection, and shorter recovery periods when compared to open appendectomy. As such, the laparoscopic approach should be attempted when considered prudent. Of note, Walsh et al. found a higher rate of fetal loss when pregnant patients underwent a laparoscopic approach, as compared to an open approach. Thus, open appendectomy appears to be a safer option for pregnant patients. Figure A shows a markedly dilated appendix with thickened wall and adjacent fat stranding, classic for acute appendicitis. Incorrect Answers: Answer 1: This is the described pathophysiology of intussusception Answer 2: This is the described pathophysiology of ovarian torsion Answer 3: This is the described pathophysiology of an ectopic pregnancy Answer 5: This is the described pathophysiology of mesenteric lymphadenitis

A 29-year-old man presents to the emergency room with severe abdominal pain. He states that for the entire day, he has had pain in his lower right abdomen in addition to a loss of appetite accompanied by nausea and vomiting. His temperature is 101.3°F (38.5°C), blood pressure is 125/98 mmHg, pulse is 78/min, and respirations are 15/min. On physical examination, he exhibits increased abdominal pain in his right lower quadrant upon deep palpation of the left lower quadrant. What is the next step in the management of this patient? Abdominal radiograph Abdominal ultrasound Colonoscopy Laparoscopic surgery Stool ova and parasite examination

D, Appendicitis This patient with acute right lower quadrant pain, fever, nausea, vomiting, and a positive Rovsing sign has appendicitis, which requires laparoscopic surgery to remove the appendix. Appendicitis has increased frequency in patients 10-30 years of age and is common in the American population due to a low fiber diet. Symptoms of appendicitis include a loss of appetite, nausea, vomiting, fever, and pain that migrates from the periumbilical area to the right lower quadrant. Physical exam may reveal rebound tenderness at McBurney point, the point over the right side of the abdomen that is one-third of the distance from the anterior superior iliac spine to the umbilicus (the location of the base of the appendix), a positive psoas sign (when the patient lies on his or her left side and pain is felt when the examiner holds the patient's right thigh and passively extends the hip), a positive obturator sign (when the patient lies on his or her back with the hip and knee flexed at 90 degrees and pain is felt upon flexion and internal rotation of the hip), and a positive Rovsing sign (when pressure placed over the descending colon on the left lower quadrant causes pain in the right lower quadrant). Incorrect Answers: Answer 1: Abdominal radiograph would not be the most appropriate step in management for this patient, as it is not a specific diagnostic test, and the patient is presenting with clear, characteristic signs suggesting appendicitis. Answer 2: Abdominal ultrasound would not be the most appropriate step in management for this patient, as the patient has characteristic signs of appendicitis. Abdominal ultrasound may be performed in pediatric and pregnant patients to spare ionizing radiation if the diagnosis of appendicitis was uncertain. Answer 3: Colonoscopy would not be an appropriate step in management in this patient with acute appendicitis. Answer 5: A stool ova and parasites diagnostic lab test would not be necessary in this patient with classic signs of acute appendicitis. Bullet Summary: Acute appendicitis is diagnosed clinically and requires laparoscopic removal of the appendix.

A 25-year-old man presents to his gastroenterologist for trouble swallowing. The patient states that whenever he eats solids, he regurgitates them back up. Given this patient's suspected diagnosis, the gastroenterologist performs a diagnostic test. Several hours later, the patient presents to the emergency department with chest pain and shortness of breath. His temperature is 99.5°F (37.5°C), blood pressure is 130/85 mmHg, pulse is 60/min, respirations are 12/min, and oxygen saturation is 99% on room air. On physical exam, the patient demonstrates a normal cardiopulmonary exam. His physical exam demonstrates no tenderness of the neck, a normal oropharynx, palpable crepitus above the clavicles, and minor lymphadenopathy. Which of the following is the best next step in management? Barium swallow Urgent surgery Magnetic resonance imaging Gastrografin swallow Ultrasound

D, Boerhaave Syndrome This patient is presenting with chest pain and palpable crepitus after endoscopy suggesting a diagnosis of esophageal perforation. The best next step in management is a gastrografin swallow. Esophageal perforation presents with severe chest pain and palpable crepitus typically occurring after endoscopy or repeat forceful vomiting. The best initial step in management is a water-soluble contrast image (gastrografin), though a CT scan or radiograph may also be appropriate initially. Once the diagnosis is confirmed, surgical correction is necessary. Incorrect Answers: Answer 1: Barium swallow would be inappropriate as it is an irritating substance and could travel out of the perforated esophagus. This substance is only used in rare cases when gastrografin is not revealing, per the discretion of the surgeon. Answer 2: Urgent surgery would be necessary once the diagnosis is confirmed. Answer 3: MRI would appropriately diagnose the condition but acquiring such imaging would take too long for such an urgent presentation. Answer 5: Ultrasound is inappropriate as it does not demonstrate the location of the perforation, and the information obtained is dependent on the habitus of the patient and the operator. Bullet Summary: The best initial step in the management for esophageal perforation is a gastrografin swallow followed by surgery.

A patient is brought to the emergency department by his spouse. The patient is admitted, treated, and transferred to the floor. The patient has a past medical history of obesity, type II diabetes mellitus, asthma, depression, and multiple suicide attempts. His current medications include pantoprazole, insulin, metformin, albuterol, fluticasone, lithium, levothyroxine, and fluoxetine. The patient has a 20 pack-year smoking history. On the floor, his temperature is 99.5°F (37.5°C), blood pressure is 140/85 mmHg, pulse is 70/min, respirations are 13/min, and oxygen saturation is 98% on room air. The patient's initial radiograph is reviewed as seen in Figure A. Which of the following is the most likely etiology of this patient's initial presentation? ANCA-positive bowel disease Bleeding arteriovenous malformation Impacted gallstone Infected mucosal outpouching Splenic laceration

D, Diverticulitis This patient is presenting with free air under the diaphragm on radiography suggesting a diagnosis of a perforated viscus. Diverticulitis (infected mucosal outpouchings of the sigmoid colon) can perforate and lead to this presentation. Perforation of the bowel presents with sudden and severe abdominal pain, tenseness upon palpation, and rebound tenderness. The best initial step in management is radiography which will reveal free air under the diaphragm. The best next step in management is antibiotics (metronidazole and ciprofloxacin), IV fluids, and emergency surgery. A perforated viscus can occur secondary to many conditions including diverticulitis, ischemic bowel, appendicitis, peptic ulcer disease, and trauma. Figure A demonstrates free air under the diaphragm which typically occurs secondary to perforated bowel (it can also be a normal finding after an abdominal surgical procedure). Illustration A demonstrates with red arrows where free air under the diaphragm can be seen. Incorrect Answers: Answer 1: ANCA-positive bowel disease describes ulcerative colitis which could be a risk factor for bowel perforation in severe presentations such as toxic megacolon (though this is less commonly associated with perforation). Other radiographic signs would likely be seen such as the lead pipe sign. Answer 2: Bleeding arteriovenous malformation describes angiodysplasia. Angiodysplasia is a common cause of lower GI bleeding but not a common cause of intestinal perforation. Answer 3: Impacted gallstone describes biliary colic. Though biliary colic or acute cholecystitis could be a risk factor for intestinal perforation, this is a less likely risk factor than acute diverticulitis. Answer 5: Splenic laceration is a surgical emergency; however, a splenic laceration does not typically present with free air under the diaphragm. Free fluid would be seen in the abdomen on FAST exam. Bullet Summary: Free air under the diaphragm is a surgical emergency and represents a likely intestinal perforation.

A 68-year-old man presents to the emergency department with left lower quadrant abdominal pain and fever for 1 day. He states during this time frame he has had a decreased appetite. The patient had surgery for a ruptured Achilles tendon 1 month ago and is still recovering but is otherwise generally healthy. His temperature is 102°F (38.9°C), blood pressure is 154/94 mmHg, pulse is 90/min, respirations are 15/min, and oxygen saturation is 98% on room air. Physical exam is remarkable for an uncomfortable man with left lower quadrant abdominal tenderness without rebound findings. Fecal occult test for blood is positive. Laboratory studies are ordered as seen below. Hemoglobin: 10 g/dL Hematocrit: 30% Leukocyte count: 13,500/mm^3 Platelet count: 157,000/mm^3 Which of the following is the most appropriate next step in management? Ceftriaxone and metronidazole Ciprofloxacin and metronidazole Colonoscopy CT abdomen MRI abdomen

D, Diverticulitis This patient is presenting with left lower quadrant abdominal pain and fever which is concerning for diverticulitis. The most appropriate next step in management is a CT of the abdomen with IV contrast to confirm the diagnosis. Acute diverticulitis typically presents in an elderly patient with a history of diverticulosis. Presenting symptoms can include decreased oral intake, fever, left lower quadrant abdominal pain/tenderness, and anorexia. Any patient with a suspected diagnosis of diverticulitis should receive a CT of the abdomen. Once the diagnosis is confirmed, the patient should be treated with antibiotics. Incorrect Answers: Answers 1-2: Ceftriaxone and metronidazole and ciprofloxacin and metronidazole are both appropriate antibiotic regimens for treating diverticulitis once the diagnosis has been confirmed with a CT. In this patient, it would be better to avoid ciprofloxacin given this patient has a ruptured Achilles tendon that was just repaired surgically. Answer 3: Colonoscopy is contraindicated in the management of diverticulitis as it could perforate a diverticulum. It is concerning that this patient has weight loss and anemia. For this reason, he should undergo a colonoscopy at some point after this infection is treated to rule out malignancy. Answer 5: MRI abdomen is unnecessary for making the diagnosis of diverticulitis and the preferred imaging modality is a CT. MRI could be used in the diagnosis of appendicitis in a pregnant woman. Bullet Summary: The most appropriate initial step in management in diverticulitis is a CT scan of the abdomen.

A 67-year-old man presents to the emergency department with abdominal pain. He states that his pain has been gradually worsening and used to be exacerbated by meals, but now occurs at rest. He also endorses diarrhea and notes blood in his bowel movements at times. He has a past medical history of obesity, diabetes, and hypertension and is generally not compliant with his medications. He was recently treated for sinusitis with amoxicillin. His temperature is 99.0°F (37.2°C), blood pressure is 157/98 mmHg, pulse is 120/min, respirations are 17/min, and oxygen saturation is 98% on room air. Physical exam is notable for generalized abdominal discomfort to palpation. Laboratory values are notable for the findings below. Hemoglobin: 12 g/dL Hematocrit: 36% Leukocyte count: 17,500/mm^3 Platelet count: 197,600/mm^3 Serum: Na+: 141 mEq/L Cl-: 100 mEq/L K+: 4.3 mEq/L HCO3-: 20 mEq/L BUN: 20 mg/dL Glucose: 99 mg/dL Creatinine: 1.5 mg/dL Ca2+: 10.2 mg/dL A CT scan of the abdomen is performed as seen in Figure A. Which of the following is the most likely diagnosis? Clostridium difficile Crohn disease Infectious diarrhea Ischemic colitis Ulcerative colitis

D, Ischemic Colitis This patient is presenting with risk factors such as diabetes with abdominal pain, bloody diarrhea, a lactic acidosis, and a CT scan demonstrating bowel wall thickening suggesting a diagnosis of ischemic colitis. Ischemic colitis occurs when the blood supply to the colon is limited. Symptoms can be exacerbated by times when the colon has increased metabolic demand such as when the patient is eating. Risk factors for this pathology include any vascular risk factors such as diabetes, hypertension, smoking, or peripheral vascular disease. Patients will typically present with generalized abdominal pain/discomfort, bloody diarrhea, a lactic acidosis, and a more critical/ill presentation than most other abdominal pathologies. The treatment of ischemic colitis is to keep the patient NPO, administer IV fluids, IV antibiotics, and surgical resection if the bowel is necrotic (such as in acute mesenteric ischemia). The most commonly affected area is the watershed area at the splenic flexure due to weak collateral circulation across the marginal artery of Drummond. Figure/Illustration A is a CT scan of the abdomen demonstrating bowel wall thickening (red arrows) which can be seen in ischemic colitis. Incorrect Answers: Answer 1: Clostridium difficile diarrhea can occur after a patient takes antibiotics and presents with profuse, watery diarrhea. It would not present with a lactic acidosis, severe abdominal pain, and bloody diarrhea. Answer 2: Crohn disease presents with abdominal pain, guaiac positive stools, and gastrointestinal lesions that can occur from mouth to anus. It usually does not present with grossly bloody stools and is more likely to present with flares versus this patient's chronic and worsening presentation. Answer 3: Infectious diarrhea presents with a self-limited episode of a fever with bloody or purulent diarrhea. It is not typically associated with changes on CT and such severe symptoms as seen in this patient. Healthy patients require no treatment and immunocompromised or critically ill patients can be treated with antibiotics. Answer 5: Ulcerative colitis presents similarly to Crohn disease but has unique findings of grossly bloody stools and symptoms/pathology limited to the colon. It would be unusual for an elderly patient to suddenly develop Crohn disease or ulcerative colitis so late in life with a gradually worsening picture. Bullet Summary: Ischemic colitis presents in a patient with vascular risk factors with symptoms including severe abdominal pain/tenderness, bloody diarrhea, a lactic acidosis, and a thickened bowel with pneumatosis coli on CT.

A 74-year-old woman with a past medical history of hypertension, peripheral artery disease, and migraine headaches presents to the emergency department with a two hour history of severe abdominal pain. The patient cannot recall any similar episodes, although she notes occasional abdominal discomfort after eating. She describes the pain as sharp periumbilcal pain. She denies recent illness, fever, chills, nausea, vomiting, or diarrhea. Her last normal bowel movement was yesterday evening. Her temperature is 37.1°C (98.8°F), pulse is 110/min, blood pressure is 140/80 mmHg, and respirations are 20/min. On exam, the patient is grimacing and appears to be in significant discomfort. Heart and lung exams are within normal limits. The patient's abdomen is soft and non-distended with diffuse periumbilical pain on palpation. There is no rebound tenderness or guarding, and bowel sounds are present. The rest of the exam is unremarkable. Labs in the emergency room show: Serum: Na+: 144 mEq/L Cl-: 105 mEq/L K+: 3.7 mEq/L HCO3-: 20 mEq/L BUN: 15 mg/dL Glucose: 99 mg/dL Creatinine: 1.2 mg/dL Ca2+: 10.7 mg/dL Phosphorus: 5.2 mg/dL Lactate: 7.0 mmol/L Amylase: 240 U/L Hemoglobin: 13.4 g/dL Hematocrit: 35% Leukocyte count: 12,100 cells/mm^3 with normal differential Platelet count: 405,000/mm^3 What is the next best step in diagnosis? D-dimer level Plain abdominal radiograph Exploratory laparotomy CT angiography Abdominal duplex ultrasound

D, Mesenteric Ischemia This elderly patient presents with a leukocytosis, metabolic acidosis, and periumbilical pain out of proportion to exam, which is most concerning for acute mesenteric ischemia (AMI). The next best step in diagnosis of AMI is CT angiography of the mesenteric vessels. AMI can result from thrombosis or embolism to the mesenteric vessels. Acute mesenteric arterial occlusion accounts for the majority of cases of AMI, with occlusion of the superior mesenteric artery being the most common site. The early clinical presentation of mesenteric ischemia is characterized by abdominal pain out of proportion to exam. Lab findings may include leukocytosis, metabolic acidosis with elevated lactate, and elevated phosphate and amylase levels. CT angiography is the modality of choice to diagnose acute mesenteric ischemia. Immediate management includes gastrointestinal decompression, fluid resuscitation, anticoagulation, and antibiotics. Incorrect Answers: Answer 1: Serum D-dimer levels that are normal can be used to help exclude acute mesenteric ischemia, but an elevated D-dimer is not sufficiently specific to aid in diagnosis of AMI. Imaging with CT angiography would still be required. Answer 2: Plain abdominal radiography may show signs such as distended loops of bowel, bowel wall thickening, or pneumatosis intestinalis. However, this test is not sufficiently sensitive to definitively rule out acute mesenteric ischemia. Answer 3: Exploratory laparotomy is not immediately indicated in this hemodynamically stable patient. Although this patient will likely require laparotomy for treatment of AMI, it is preferable to first establish a definitive diagnosis via CT angiography. Answer 5: Abdominal ultrasound is used to aid in the diagnosis of many abdominal conditions. For patients with known mesenteric thrombosis that has been treated with a stent, duplex ultrasound can be used for monitoring stent function. It is not the imaging modality of choice in diagnosis. Bullet Summary: Acute mesenteric ischemia presents with periumbilical abdominal pain out of proportion to exam, leukocytosis, and metabolic acidosis; diagnosis is confirmed with mesenteric angiography followed by laparotomy.

A 47-year-old woman presents to the emergency department with abdominal pain. The patient states that she felt this pain come on during dinner last night. Since then, she has felt bloated, constipated, and has been vomiting. Her current medications include metformin, insulin, levothyroxine, and ibuprofen. Her temperature is 99.0°F (37.2°C), blood pressure is 139/79 mmHg, pulse is 95/min, respirations are 12/min, and oxygen saturation is 98% on room air. On physical exam, the patient appears uncomfortable. Abdominal exam is notable for hypoactive bowel sounds, abdominal distension, and diffuse tenderness in all four quadrants. Cardiac and pulmonary exams are within normal limits. Which of the following is the best next step in management? Emergency surgery IV antibiotics and steroids Metoclopramide Nasogastric tube, NPO, and IV fluids Stool guaiac

D, Small Bowel Obstruction This patient is presenting with abdominal pain, vomiting, decreased bowel sounds, distension, and diffuse abdominal tenderness suggesting a diagnosis of small bowel obstruction. The best initial step in management is placement of a nasogastric tube, NPO, and IV fluids. Small bowel obstruction typically occurs in patients with a history of bowel surgery and occurs due to entrapment of the bowel in adhesions. It presents with moderate/severe colicky abdominal pain, vomiting, absence of flatus/stools, distension, and diffuse abdominal tenderness. Radiography will often reveal distension of the bowel prior to the transition point (the location where the bowel is obstructed). The best initial step in management is placement of a nasogastric tube (for decompression of the bowel), NPO, and IV fluids followed by surgery if symptoms are persistent/ischemia is suspected. Incorrect Answers: Answer 1: Emergency surgery is indicated in small bowel obstruction if symptoms are refractory to initial decompressive therapy or if symptoms worsen/bowel perforation is suspected. Answer 2: IV antibiotics and steroids are the best initial step in management for toxic megacolon which can occur in the setting of inflammatory bowel disease, ischemia, or infections such as C. difficile. Answer 3: Metoclopramide is a gastroprokinetic that could be used to treat diabetic gastroparesis, which would present in a diabetic patient with constipation, bloating, and potentially nausea/vomiting. However, this would be a chronic presentation versus an acute presentation. Answer 5: Stool guaiac would not add any information to the diagnosis given the patient's presentation. It could be performed at a later time after the initial interventions are started. Bullet Summary: The best initial step in management in small bowel obstruction is placement of a nasogastric tube, NPO, and IV fluids.

A 69-year-old female presents to the emergency department with crampy abdominal pain. She has a past medical history of hypertension, dyslipidemia, and cholelithiasis status post cholecystectomy. The patient states she has not passed stool or gas for 48 hrs. On physical exam vitals are T 98.4 F HR 105 bpm BP 155/101 mmHg RR 16 SpO2 96%, abdominal exam is notable for distension, tympany to percussion, and tenderness to palpation without rebound or guarding. Which of the following findings are most likely on radiograph? Free air under the diaphragm on upright abdominal radiograph Apple core defect after lower GI series Multiple punctate mucosal outpouchings through the serosa on lower GI series Ladder-like series of distended bowel loops with air-fluid levels on abdominal radiograph Loss of haustral markings and "lead pipe" appearance to the descending colon on lower GI series

D, Small Bowel Obstruction This patient with prior abdominal surgery presenting with a painful, distended abdomen and history of obstipation is highly suggestive of an acute small bowel obstruction. Abdominal radiograph is most likely to show Ladder-like series of distended bowel loops with air-fluid levels (Illustration A). Small bowel obstructions account for around 15% of all surgical admissions in the United States. Adhesive disease, typically secondary to previous surgery, accounts for around 75% of all small bowel obstructions, with malignancy and hernias accounting for another total 16%. Conservative management with nasogastric suction, IV fluid hydration, and bowel rest is an appropriate first strategy for patients who present without evidence of peritonitis and who are hemodynamically stable. Patients who fail conservative management or who present with more advanced disease require immediate surgical management. Jackson et al. discuss the diagnosis and management of intestinal obstructions in adults. They conclude that in the hemodynamically stable patient, IV fluid hydration, nasogastric intestinal decompression, and bowel rest is an appropriate first strategy to attempt to resolve the obstruction. However, hemodynamically unstable patients, patients with perforations or ischemic gut, and patients who fail conservative management require surgical intervention. Maung et al. discuss the diagnostic approach to patients with small bowel obstructions. They conclude that the use of helical computed tomography plays a central role to the diagnosis of small bowel obstructions, as it provides incremental clinically relevant information over plain films that may lead to changes in management. Illustration A shows a partially upright abdominal radiograph with dilated loops of small bowel and air fluid levels, consistent with a small bowel obstruction. Incorrect Answers: Answer 1: Free air under the diaphragm is characteristic of hollow viscus perforation. This is unlikely given this patient's abdominal exam and vital status. Answer 2: This is a characteristic finding of colon cancer, this patient's presentation is more consistent with a small bowel obstruction. Answer 3: This is a characteristic finding in diverticulosis, not small bowel obstruction. Answer 5: This is a characteristic finding in ulcerative colitis.

A 3-week-old male presents to the emergency department with vomiting. His parents report that he suddenly started vomiting this morning and has shown no interest in feeding since then. They describe the vomitus as green and without any traces of blood. Prior to today, the patient was feeding, voiding and stooling well. He was noted to have surpassed his birth weight at his last office visit one week ago. In the emergency room, his temperature is 97.6°F (36.4°C), blood pressure is 78/56 mmHg, pulse is 148/min, and respirations are 44/min. On physical exam, the patient is in mild distress. He has no dysmorphic features, and his mucous membranes are dry. His abdomen is soft and distended. Bowel sounds are hypoactive. An abdominal radiograph is performed and can be seen in Figure A. Which of the following is the best next step in the diagnosis of this condition? Abdominal CT Abdominal ultrasound Contrast enema Upper gastrointestinal series Rectal suction biopsy

D, Volvulus This neonatal patient presents with sudden-onset bilious vomiting and an abdominal radiograph revealing a gasless abdomen, which is consistent with intestinal malrotation complicated by a midgut volvulus. The best next step in diagnosis of intestinal malrotation is an upper gastrointestinal series. Malrotation classically presents with bilious vomiting due to formation of a midgut volvulus. Findings of malrotation on upper gastrointestinal series include an abnormally placed ligament of Treitz on the right side of the abdomen and a "corkscrew" appearance of the distal duodenum (Illustration A). If upper gastrointestinal series confirms the diagnosis, patients should undergo surgery to reduce the volvulus. Figure A demonstrates the nonspecific finding of a paucity of air in the abdomen on abdominal radiograph, which is consistent with intestinal malrotation complicated by midgut volvulus. Illustration A demonstrates the classic findings of intestinal malrotation and midgut volvulus on contrast enema, which includes visualization of the ligament of Treitz on the right side of the abdomen, and a "corkscrew" pattern of contrast as it fills the distal duodenum and proximal jejunum. Incorrect Answers: Answer 1: Although abdominal CT would detect intestinal malrotation, CT should be avoided in the pediatric population in light of the radiation exposure. Answer 2: Abdominal ultrasound is the test of choice for diagnosing pyloric stenosis, which presents with nonbilious vomiting and a palpable olive mass in the abdomen. Answer 3: A contrast enema is used to differentiate between Hirschsprung disease and meconium ileus but would not be helpful in diagnosing malrotation as the midgut volvulus occurs in the small intestine Both Hirschsprung disease and meconium ileus present in neonatal patients with bilious vomiting and failure to pass meconium in the first 48 hours of life. Answer 5: Rectal suction biopsy is the most accurate test to diagnose Hirschsprung disease. Although Hirschsprung disease commonly presents in neonatal patients with bilious vomiting, an abdominal radiograph in Hirschsprung disease would likely show dilation of the proximal colon. Bullet Summary: The best diagnostic test in malrotation in an infant is upper gastrointestinal series, which will show abnormal placement of the duodenum and ligament of Treitz on the right side of the abdomen.

A 74-year-old female presents to the emergency room with abdominal pain. She notes several days of constipation and most recently has not had a bowel movement in 3 days. She is unable to remember if she has passed gas. Her vital signs are BP 118/76 mmHg, HR 97 beats per minute, RR 17 breaths per minute, and T 99.4 degrees Fahrenheit, and her abdomen is noticeably distended. A barium enema radiograph is shown in Figure A. Which of the following is the best next step? Manual evacuation of rectal impaction Administer large doses of laxatives Laparotomy with sigmoid resection Proctosigmoidoscopy and placement of a rectal tube Observation

D, Volvulus This patient's clinical presentation is consistent with sigmoid volvulus. The first-line treatment for this condition is proctosigmoidoscopy and placement of rectal tube. Elderly patients presenting with abdominal pain and constipation should be evaluated for volvulus. On barium enema radiograph, sigmoid volvulus is associated with a "parrot's beak" sign, which is characterized by a gas shadow in the right upper quadrant that tapers towards the left lower quadrant. The mortality rate for sigmoid volvulus is up to 45%, usually resulting from delayed diagnosis. Figure/Illustration A shows a barium enema radiograph typical of a sigmoid volvulus, with notable parrot's beak sign (yellow arrow). Incorrect Answers: Answer 1: This is the treatment for rectal impaction, which does not present with a parrot beak sign on X-ray. Answer 2: Regular use of laxatives is associated with a higher risk of sigmoid volvulus, and acute laxative treatment is not thought to be beneficial in patients with a sigmoid volvulus. Answer 3: Laparotomy with sigmoid resection should be performed either electively after decompression or in the case of unsuccessful decompression. Answer 5: This patient's condition carries up to a 40% mortality rate, and her bowel should be decompressed immediately.

A 75-year-old man presents to the emergency department for abdominal pain. The patient states the pain started this morning and has been worsening steadily. He decided to come to the emergency department when he began vomiting. The patient has a past medical history of obesity, diabetes, alcohol abuse, and hypertension. His current medications include captopril, insulin, metformin, sodium docusate, and ibuprofen. His temperature is 104.0°F (40°C), blood pressure is 160/97 mmHg, pulse is 90/min, respirations are 15/min, and oxygen saturation is 98% on room air. Abdominal exam reveals left lower quadrant tenderness. Cardiac exam reveals a crescendo systolic murmur heard best by the right upper sternal border. Lab values are ordered and return as below. Hemoglobin: 15 g/dL Hematocrit: 42% Leukocyte count: 19,500 cells/mm^3 with normal differential Platelet count: 226,000/mm^3 Serum: Na+: 139 mEq/L Cl-: 101 mEq/L K+: 4.4 mEq/L HCO3-: 24 mEq/L BUN: 22 mg/dL Glucose: 144 mg/dL Creatinine: 1.2 mg/dL Ca2+: 9.8 mg/dL Which of the following is the most accurate test for this patient's condition? Amylase and lipase levels Barium enema Colonoscopy CT scan Sigmoidoscopy

D,Diverticulitis This patient is presenting with left lower quadrant abdominal pain, fever, and a leukocytosis suggesting a diagnosis of diverticulitis. The most accurate test for diverticulitis is a CT scan. Diverticulitis presents with left lower quadrant abdominal pain, systemic symptoms (such as fever), nausea, vomiting, and a leukocytosis on lab values. It typically occurs in a patient with a history of diverticulosis. The most accurate test for diverticulitis is a CT scan. Colonoscopy is contraindicated as it could cause rupture. Further management is keeping the patient NPO and administering ciprofloxacin, metronidazole, and IV fluids. Patients who can not tolerate oral antibiotics should be started on IV antibiotics. Incorrect Answers: Answer 1: Amylase and lipase levels are specific lab values for pancreatitis (mostly lipase). Pancreatitis presents with nausea, vomiting, and epigastric pain in an alcoholic. Answer 2: Barium enema is contraindicated in diverticulitis as it could cause irritation. A barium enema is the most accurate test in diverticulosis. Answers 3 & 5: Colonoscopy and sigmoidoscopy are contraindicated in diverticulitis as they could cause rupture. Colonoscopy is an accurate test for diverticulosis. Bullet Summary: The most accurate test for diverticulitis is a CT scan.

A 65-year-old man with a long history of constipation presents with bright red blood per rectum for 1 day. He denies any straining, abdominal pain, diarrhea, or lightheadedness. He denies any recent trauma and family history is unremarkable. His last colonoscopy was about 12 years ago and patient reports he had some "blebs" in his colon. Physical exam was largely unremarkable and a rectal exam did not show any perianal fissures or hemorrhoids. Stool hemocult test is positive.

Diverticular Disease (Diverticulitis)(snapshot)

A 63-year-old female is brought to the emergency room for severe abdominal pain for the past 1 day. She reports that the pain is 9/10, dull, with intermittent exacerbations concentrated at the left lower quadrant (LLQ). She endorses low-grade fever, nausea, and vomiting for the past couple of hours. Her past medical history includes hypertension, constipation, and diverticulosis. A physical examination demonstrates abdominal tenderness and some diffuse voluntary guarding.

Diverticulitis (snapshot)

A 61-year-old man presents to the clinic for an annual wellness exam. He denies any specific concerns except for some constipation for the past week. His past medical history is significant for hypertension that is controlled with lisinopril. He denies fever, weight changes, melena, hematochezia, abdominal pain, or stool changes. He subsequently undergoes a routine colonoscopy which demonstrates small outpouchings within the sigmoid colon.

Diverticulosis (snapshot)

A 42-year-old woman presents to the emergency department with abdominal pain. She states that she was eating dinner when she suddenly felt abdominal pain and nausea. The pain did not improve after 30 minutes, so her husband brought her in. The patient has a past medical history of diabetes that is well-treated with exercise and metformin. Her temperature is 101°F (38.3°C), blood pressure is 147/98 mmHg, pulse is 90/min, respirations are 17/min, and oxygen saturation is 98% on room air. Physical exam reveals right upper quadrant tenderness and guaiac negative stools. Which of the following is optimal management for this patient's condition? NPO, IV fluids, analgesics, antibiotics NPO, IV fluids, analgesics, antibiotics, emergent cholecystectomy NPO, IV fluids, analgesics, antibiotics, cholescystecomy within 24 hours NPO, IV fluids, analgesics, antibiotics, cholescystecomy within 48 hours NPO, IV fluids, analgesics, antibiotics, cholescystecomy within 72 hours

E, Acute Cholecystitis This patient is presenting with symptoms of acute cholecystitis and should receive NPO orders, IV fluids, analgesics, antibiotics, and a cholescystectomy within 72 hours. Acute cholecystitis presents with right upper quadrant abdominal pain, a positive Murphy sign, fever, and a leukocytosis. NPO, IV fluids, analgesics, antibiotics, and cholescystectomy within 72 hours are the appropriate management of this condition. Given poor patient dietary compliance, patients often receive the procedure within the same hospital stay rather than being discharged with outpatient followup. Prior to surgery, the diagnosis should be confirmed with an ultrasound or a CT scan. Incorrect Answers: Answers 1-4: The optimal time frame for cholecystectomy is within 72 hours. Due to poor patient compliance, it is often recommended that the patient have the procedure done while in the hospital or else relapse is very common. Bullet Summary: NPO, IV fluids, analgesics, antibiotics, and cholescystecomy within 72 hours is the optimal treatment of acute cholecystitis.

A 55-year-old man is brought to the emergency department by police. The patient was found unconscious in a dumpster. He has a past medical history of HIV and alcohol abuse and is not currently taking any medications. His temperature is 97.5°F (36.4°C), blood pressure is 100/67 mmHg, pulse is 98/min, respirations are 15/min, and oxygen saturation is 97% on room air. On physical exam, the patient is now coherent and responsive. He complains of pain in his abdomen that is severe. He says the pain radiates to his back. On physical exam, the patient has notable epigastric tenderness. The patient's cardiac exam is within normal limits. His pulmonary exam reveals mild bibasilar crackles. A CT scan is ordered as seen in Figure A. Which of the following is the next best step in management? Abdominal radiograph Abdominal ultrasound Ciprofloxacin and metronidazole Emergency surgery NPO, IV fluids, NG tube

E, Acute Pancreatitis This patient is presenting with alcohol intoxication, epigastric pain, and CT scan suggestive of acute pancreatitis. The best initial step in the management of acute pancreatitis is to keep the patient NPO, administer IV fluids, and place an NG tube. Acute pancreatitis occurs in obese patients after a fatty meal, patients with gallstones, or in alcoholics during episodes of binge drinking. Patients will present with epigastric pain that radiates to the back. Laboratory values will reveal an elevated amylase and lipase that is at least three times greater than normal baseline values. The diagnosis can be confirmed with a CT scan which will reveal edema and inflammation surrounding the pancreas. The best initial treatment is to keep the patient NPO, administer IV fluids, and place a NG tube. Figure A is a CT scan demonstrating edema and inflammation around the pancreas that would be seen in acute pancreatitis. Illustration A demonstrates the pancreas normal contour (red line) and the area of edema surrounding the pancreas (yellow crescent). Incorrect Answers: Answer 1: Abdominal radiograph would not be indicated after an abdominal CT scan was performed as a CT scan reveals more detail. An emergency radiograph could be ordered if the suspected diagnosis was a perforated viscus though this could be seen on a CT scan as well. Answer 2: Abdominal ultrasound would be indicated to diagnose gallstones or an abdominal aortic aneurysm which is a less likely diagnosis given this patient's history and CT findings. An ultrasound would likely not be performed after a CT scan for pancreatitis. Answer 3: Ciprofloxacin and metronidazole are the antibiotics of choice when the diagnosis is a perforated viscus, diverticulitis, or ascending cholangitis. Answer 5: Emergency surgery is not indicated in pancreatitis but would be indicated if the diagnosis was a perforated viscus. Bullet Summary: Acute pancreatitis is best managed with keeping the patient NPO, administering IV fluids, and placing an NG tube.

A 33-year-old man with a history of alcohol abuse and cirrhosis presents to the emergency department with profuse vomiting. The patient is aggressive, combative, emotionally labile, and has to be chemically restrained. The patient continues to vomit and blood is noted in the vomitus. His temperature is 99.2°F (37.3°C), blood pressure is 139/88 mmHg, pulse is 106/min, respirations are 17/min, and oxygen saturation is 100% on room air. The patient complains of sudden onset chest pain during his physical exam. A crunching and rasping sound is heard while auscultating the heart. Which of the following is the pathophysiology of the most likely diagnosis? Dilated and tortuous veins Inflammation of the pericardium Mucosal tear Pericardial fluid accumulation Transmural tear

E, Boerhaave Syndrome This patient is presenting with profuse, bloody emesis with Hamman sign (crunching heart sounds) which is concerning for esophageal rupture (Boerhaave syndrome) which is due to a transmural tear of the esophagus. Esophageal rupture can occur secondary to a procedure (such as endoscopy) or after profuse vomiting causes trauma to the esophagus leading to a transmural tear. Esophageal rupture will present with severe pain, dysphagia, chest pain, subcutaneous emphysema, and possibly Hamman sign which is a rasping/crunching sound heard when auscultating the heart secondary to mediastinal air. After the patient is stabilized, the diagnosis is confirmed with a gastrografin swallow and possible surgical repair of the tear. Incorrect Answers: Answer 1: Dilated and tortuous veins are the pathophysiology of bleeding esophageal varices. While bleeding esophageal varices are possible in this patient with a history of cirrhosis, they would not be associated with Hamman sign. Answer 2: Inflammation of the pericardium describes pericarditis which presents with chest pain relieved by leaning forward and worsened by leaning back, a friction rub, and diffuse ST-elevations with PR depression on ECG. Answer 3: Mucosal tear describes a Mallory-Weiss tear which presents with bloody vomitus but would not present with severe pain and Hamman sign. Answer 4: Pericardial fluid accumulation describes cardiac tamponade which presents with hypotension, tachycardia, jugular venous distension, muffled heart sounds, and electrical alternans on ECG. Bullet Summary: Boerhaave syndrome occurs after profuse vomiting and occurs secondary to a transmural tear of the esophagus.

A 56-year-old man presents to the emergency department with severe epigastric pain that began an hour prior to presentation. He describes the pain as sharp, 10/10 in severity, and radiating to the back. Swallowing worsens the pain and causes him to cough. Before the pain started, he had been vomiting multiple times per day for the past week. The emesis was yellow and he denied ever seeing blood. Medical history is significant for poorly controlled hypertension, type II diabetes, alcohol use disorder, and 2 prior hospitalizations for acute pancreatitis. He smokes 1 pack of cigarettes per day for the last 35 years, denies illicit drug use, and drinks 3 pints of vodka per day. On physical exam, there is mediastinal crackling in synchrony with cardiac contraction on cardiac auscultation in the left lateral decubitus position. Laboratory testing is significant for leukocytosis. Which of the following is most likely the cause of this patient's symptoms? Coronary artery occlusion Dissection of the aorta Inflammation of the pancreas Ulcerative changes in the gastric mucosa Transmural esophageal rupture

E, Boerhaave Syndrome This patient's severe epigastric pain, Hamman sign, and continuous vomiting is suggestive of Boerhaave syndrome (BS). BS results from transmural rupture of the esophagus. Boerhaave syndrome, also known as effort rupture of the esophagus, is a type of esophageal perforation secondary to acute elevations in intraesophageal pressure. Typically, patients will have a history of retching and vomiting prior to symptom development. On physical exam there may be crepitus upon palpation of the chest wall secondary to subcutaneous emphysema. There may also be mediastinal crackling concurrent with cardiac contractions (Hamman sign). The diagnosis is based on history, physical, and contrast esophagram or computed tomography (CT) of the chest (and abdomen, if necessary). Boerhaave syndrome is a surgical emergency. Incorrect Answers: Answer 1: Coronary artery occlusion typically presents as a pressure-like pain that radiates down the left arm or jaw. Answer 2: Dissection of the aorta presents with severe chest pain that radiates to the back. Although this patient has poorly controlled hypertension, aortic dissection would not explain his Hamman sign. Answer 3: Inflammation of the pancreas does not explain his physical exam findings. Answer 4: Ulcerative changes in the gastric mucosa can present with epigastric pain. However, it does not explain his physical exam findings. Bullet Summary: Boerhaave syndrome results from transmural rupture of the esophagus.

A 26-year-old African American woman presents to the clinic with burning upon urination. The patient describes increased frequency, urgency, and a painful sensation at the urethra when urinating for the past 3 days. She also reports increased vaginal discharge and abnormal odor during the same time. The patient denies fever, flank pain, or hematuria (though the patient does report a dark brown color of the urine). Her past medical history is significant for Crohn disease that is controlled with sulfasalazine. Vital signs are within normal limits. What is the definitive treatment of the described condition? Bactrim IM ceftriaxone and oral azithromycin Increase in sulfasalazine dose IV ceftriaxone Surgery

E, Diverticulitis The patient's dysuria, increased frequency, and urgency suggests a urinary tract infection (UTI). The "dark brown" urine in addition to the history of Crohn suggests UTI secondary to a colovesical fistula, which is definitively treated with operative management (surgery). Colovesical fistula is an abnormal connection between the colon and the bladder and is most commonly associated with diverticulitis, cancer, or Crohn disease. The condition commonly presents with pneumaturia, fecaluria, suprapubic pain, and UTI symptoms (dysuria, urgency, and frequency). Diagnosis is confirmed via abdominopelvic CT with oral/rectal contrast along with a colonoscopy if positive to rule out malignancy. Incorrect Answers: Answer 1: Bactrim is one of the first-line treatments for UTI, which would be initial management of a patient with colovesical fistula but not the definitive treatment. Answer 2: IM ceftriaxone and oral azithromycin is the treatment for chlamydial/gonorrheal infection, which may be suspected from the patient's dysuria, discharge, and odor. However, it would not present with fecaluria. Answer 3: Increase in sulfasalazine dosage may or may not be indicated in the management of the patient's Crohn disease. However, it would not definitively treat the colovesical fistula. Answer 4: IV ceftriaxone would be indicated if the patient is displaying signs of pyelonephritis such as fever, chills, CVA tenderness, sepsis, etc. Bullet Summary: The definitive treatment for a colovesical fistula, which is often a complication of diverticulitis, malignancy, or Crohn disease, is via surgical resection.

A 46-year-old man with no significant medical history presents to the emergency department with severe abdominal pain and fever. He states that earlier today, he began experiencing nausea before the sharp pain began in his umbilicus and localized to his left lower abdomen. He is febrile to 103.2°F (39.6°C), but otherwise normotensive with mild tachycardia to 100/min. Physical exam is notable for left lower quadrant pain and abdominal distension, but no involuntary guarding or rigidity. A complete blood count reveals leukocytosis with left shift. CT of the abdomen and pelvis is shown in Figure A. Which of the following is appropriate for this patient? Admit and start intravenous antibiotics Admit for emergent laparoscopic sigmoidectomy Discharge to home with referral for flexible sigmoidoscopy Discharge to home on intravenous antibiotics Discharge to home on oral antibiotics

E, Diverticulitis This patient is presenting with acute uncomplicated diverticulitis. The patient can be discharged on empiric oral antibiotics that cover both gram-negative and anaerobes (ciprofloxacin and metronidazole). Diverticulitis can be classified as either uncomplicated or complicated. Complicated diverticulitis occurs in the setting of hemorrhage, perforation, abscess, obstruction, or fistula. Uncomplicated diverticulitis involves bowel wall inflammation and thickening only. Diagnosis is most commonly made with computed tomography. Patients with mild uncomplicated diverticulitis can be treated as outpatients with clear liquid diets and broad spectrum oral antibiotics such as ciprofloxacin and metronidazole to cover gram-negative bacteria and anaerobes. This has been supported by the DIVER trial and endorsed by the American College of Gastroenterology and the American Gastroenterology Association (1). Patients should be hospitalized if complicated disease is evident, if the patient is septic, if the patient cannot tolerate oral intake, or if the patient lacks social support at home. Figure A shows a CT scan of a patient with diverticulitis. Note the circle highlighting the thickened colonic wall and the absence of intraperitoneal free air/fluid. Illustration A reveals the sigmoid colon with multiple diverticula. Incorrect Answers: Answer 1-2: Admission is not necessary for this patient who otherwise has no other comorbidities or risk factors. Answer 3: Referral for an invasive procedure once the diagnosis of diverticulitis has been confirmed is unnecessary and harmful. Answer 4: Discharge to home on intravenous antibiotics is inappropriate, at least due to lack of patient education and skill with intravenous access. Bullet Summary: Acute uncomplicated diverticulitis merits conservative outpatient management with bowel rest and empiric antibiotics against gram-negatives and anaerobes.

A 33-year-old woman with a past medical history of Crohn disease, depression, and pelvic inflammatory disease presents to the emergency department with abdominal pain, nausea, and vomiting. She states that her symptoms started yesterday and have been worsening until today. She has been unable to have a bowel movement during this time, and every time she eats she vomits shortly thereafter. Her initial vitals are unremarkable, and physical exam is notable for a distended and tympanic abdomen. One hour later, the patient complains of worsening abdominal pain. Her temperature is 97.9°F (36.6°C), blood pressure is 104/64 mmHg, pulse is 140/min, respirations are 12/min, and oxygen saturation is 98% on room air. Laboratory values are drawn at that time as seen below. Hemoglobin: 14 g/dL Hematocrit: 44% Leukocyte count: 10,500/mm^3 with normal differential Platelet count: 233,000/mm^3 Lactate: 5 mmol/L (normal < 1.0 mmol/L) The patient's exam is notable to have significant rebound tenderness. Which of the following is the most appropriate next step in management? CT abdomen/pelvis MRI abdomen/pelvis Nasogastric tube Transvaginal ultrasound Urgent surgery

E, Small Bowel Obstruction This patient has a history of abnormal bowel (given the transmural inflammation from Crohn disease) and is initially presenting with symptoms suggestive of small bowel obstruction (nausea, vomiting, and failure to have a bowel movement). Her symptoms suddenly worsen, and she experiences rebound tenderness (suggesting irritation of the peritoneum or bowel rupture) and a lactic acidosis suggesting bowel ischemia, which requires immediate treatment in the operating room. A small bowel obstruction occurs whenever there is abnormal bowel that has adhesions which subsequently entrap the bowel. Previous surgeries, trauma that violates the abdomen, or inflammatory conditions such as Crohn disease are all risk factors for small bowel obstructions. Any patient with a suspected small bowel obstruction should have a CT scan performed to confirm the diagnosis, have a nasogastric tube placed to decompress the bowel, have the pain controlled, and IV fluids administered. If there is a worsening of symptoms, unstable vitals, or any concern for perforation, the patient must immediately be taken to the operating room. Incorrect Answers: Answer 1: CT abdomen/pelvis would be the appropriate initial study for this patient with symptoms of a small bowel obstruction; however, this patient's symptoms have progressed and become more severe. Also, she is hemodynamically unstable making CT an inappropriate test. Answer 2: MRI abdomen/pelvis is not the appropriate study for diagnosing a small bowel obstruction. It may be indicated to diagnose appendicitis in a pediatric or pregnant patient as it decreases radiation exposure. Answer 3: Nasogastric tube and IV fluids are the appropriate management of a small bowel obstruction as this combination decompresses the bowl while providing IV hydration since the patient is NPO. This unstable patient should immediately be taken to the operating room. Answer 4: Transvaginal ultrasound would be the appropriate study for ovarian torsion, which presents with sudden onset of severe lower abdominal pain with nausea and vomiting. An ultrasound would show an absence of blood flow to the ovary and ovarian edema. Treatment involves urgent laparoscopy. Bullet Summary: A small bowel obstruction that is complicated (unstable vitals, lactic acidosis, or severe/worsening pain) requires urgent surgery.

A 69-year-old female presents to the emergency room for bright red blood per rectum for the past 2 hours. She noticed some blood in her stool shortly after experiencing some abdominal pain. The pain is reported as dull, achy, 5/10, and concentrated at the left lower quadrant. Her past medical history is significant for hypertension and a past myocardial infarction (MI).

Ischemic Colitis (snapshot)

A 73-year-old woman presents with severe abdominal pain and a recent history of passing bloody stools. The pain began about 3 days ago and is more concentrated at the left side of the abdomen. Physical examination shows decreased-to-absent bowel sounds, abdominal distention, and diffuse tenderness to palpation over her abdomen. A barium-enema study is performed which shows "thumb-printing" of the colon.

Ischemic Colitis (snapshot)

An 80-year-old man with a history of divertulosis and multiple episodes of diverticulitis presents to the emergency room for increasing crampy abdominal pain, nausea, and vomiting. He last had a bowel movement 4 days ago and has had tiny amounts of diarrhea. He has not had any flatus since last night. On physical exam, his abdomen is noted to be distended, somewhat rigid, and tender to palpation. Bowel sounds are absent. A digital rectal exam reveals hard stool in the rectal vault. An abdominal radiograph shows dilated loops of bowel but no free air under the diagphragm. A computed tomography of his abdomen and pelvis confirms the presence of dilated loops of bowel and a large fecolith.

LBO

A 65-year-old gentleman presents with acute onset severe abdominal pain. The pain is described as "diffuse" without localization and constant. The patient states that he has had nausea and vomitting with this episode. Physical examination shows absent bowel sounds, mild abdominal distention, and pain that is disproportionate to the exam findings.

Mesenteric Ischemia (snapshot)

A 68-year-old woman is brought to the emergency room for severe abdominal pain for the past 2 hours. She reports that she was watching TV when suddenly her stomach began hurting. The pain got progressively worse and is currently described as stabbing, 9/10 pain distributed throughout the abdominal region. Her past medical history is significant for atrial fibrillation.

Mesenteric Ischemia (snapshot)

A 50-year-old woman presents to the emergency room for nausea, vomiting, loss of appetite, and abdominal pain. She has a history of Crohn disease, on infliximab, but she reports that this did not feel like a flare of her disease. On physical exam, there is tenderness to palpation of her abdomen without any peritoneal signs. An abdominal radiograph shows dilated loops of small bowel. She is started on intravenous fluids, given anti-emetics, and put on bowel rest.

SBO

A 45-year-old woman presents to the emergency room for abdominal pain, nausea, and nonbloody vomiting for the past day. She has not had any flatus during this time. She also endorses decreased appetite. She has a past surgical history of cesarean section. On physical exam, there is diffuse tenderness to palpation of her abdomen but no rebound tenderness or guarding. An abdominal radiograph shows dilated loops of small bowel, which is confirmed with a CT abdomen and pelvis. There are no masses identified. She is admitted to the inpatient floor and a nasogastric tube is placed.

Small Bowel Obstruction (snapshot)

A 10-day-old baby boy is brought to the emergency room for repeated episodes of vomiting. His mother describes the vomit as green and yellow. On physical exam, the physician has difficulty at first locating the heartbeat, but finally hears a heartbeat on the right side of the chest. Suspicious, he sends the baby for abdominal radiography and upper GI series. Imaging reveals dilated bowel loops and a "birds beak" appearance of bowel.

Volvulus (snapshot)

A 2-day-old neonate presents with bilious vomiting and irritability. On physical examination, she has abdominal distention and high-pitched bowel sounds. Barium enema radiograph reveals a dilated sigmoid colon with a column of barium resembling a "coffee bean".

Volvulus (snapshot)


Kaugnay na mga set ng pag-aaral

Chapter 36 - Key Terms and Concepts

View Set

Marketing Ch 5-7, Commonly Missed

View Set

Growth and Lifespan Development EPPP

View Set

Chapter 32. Personalizing the Conversation: Beethoven and the Classical Sonata, Music Chapter 31, Music Chapter 30, Music Chapter 29, music test 2

View Set

CS-145 Final Exam Study (Quiz 1-8)

View Set